Obs and Gynae Flashcards

1
Q

What are the phases of the menstrual cycle

A

Follicular phase

  • Increasing FSH and LH - stimulate primary follicle growth, secretion + conversion of androgens to oestrogens and inhibin section.
  • this increases oestrogen - starts to inhibit FSH - dominant follicle grows
  • once reaches maturity oestrogen ↑↑ –> LH surge –> follicle rupture

Luteal phase

  • remaining follicle become corpus luteum
  • Progesterone ↑ - maintain conditions for fertilisation and implantation
  • if not fertilised - corpus luteum spontaneously regresses (PG cut of blood supply) - ↓ Oest & prog –> menses
  • if fertilised - syncytiotrophoblast of embryo will produce hCG –> maintain endometrium (prod progesterone)
How well did you know this?
1
Not at all
2
3
4
5
Perfectly
2
Q

Phases of the uterine cycle

A
Proliferative Phase (same time as follicular - prepare for fertilisation)
- Oestrogen 
 fallopian tube formation, thickening of endometrium, ↑ growth and motility of myometrium and produce thin alkaline cervical mucus (to facilitate sperm transport)
Secretory Phase (along luteal phase) 
- Progesterone 
 further thickening of the endometrium (glandular secretory form + development of spiral A.) and the myometrium; ↓ of motility of the myometrium (don’t want contractions); thick acidic cervical mucus production (prevent polyspermy); ↓ fallopian tube motility, secretion and cilia activity; changes in mammary tissue and other metabolic changes; elevates basal body temperature.
How well did you know this?
1
Not at all
2
3
4
5
Perfectly
3
Q

What is the menopause

A

The end of the reproductive life - the ovarian follicles are depleted and there has been amenorrhoea for 12 months.
Usually between the ages of 45 and 55y/o.
2 FSH levels >30IU/L - suggestive

How well did you know this?
1
Not at all
2
3
4
5
Perfectly
4
Q

Symptoms of menopause

A

• Menstrual irregularity - cycles become anovulatory, before stopping
• Vasomotor disturbance—sweats, palpitations, and hot flushes (peripheral vasodilation and a transient rise in body temp)
• Atrophy of oestrogen-dependent tissues (genitalia, breasts, bladder and urethra) and skin. Vaginal dryness can lead to vaginal and urinary infection, dyspareunia, traumatic bleeding, stress incontinence, and prolapse.
• Osteoporosis: ↓ oestrogen –> ↑ osteoclast activity –> acceleration of age related loss of bone density and ↑ frequency in # - femur, neck, radius and vertebrae
• Oest. is protective effect against IHD (↓ LDL + ↑ HDL)
Other: Joint stiffness/soreness; dizzy; interrupted sleep; anxiety; reduced concentration; irritable/ mood swings

How well did you know this?
1
Not at all
2
3
4
5
Perfectly
5
Q

Management of menopause

A

Lifestyle
o Stop smoking, healthy diet, exercise + weight loss,↓ stress, ↓ caffeine/spicy food/ alcohol
o Stay cool at night; Vaginal lubrication
o Annual BP, breast checks

HRT

  • oestrogen only (if had hysterectomy)
  • cyclical - if still periods –>oestrogen every day then progesterone either last 14d of -cycle(reg)/ 3m (irreg)
  • continous combined - if post-menopausal

o SSRI - for hot flushes
o Clonidine - ↓ hot flushes + night sweats (doesn’t affect hormone levels – no ↑ risk Ca)
 SE –> dry mouth, drowsiness, depression and constipation
o Tibolone – agonist at oestrogen receptor – relieve symptoms (same risks HRT)
o Raloxifene (SERM) - protects bones, ↓ breast & endometrial ca risk (ineffective for flushes)
o Natural hormones not recommended as not regulated and effect not known
o Complementary therapies (be careful about ADR)  Black cohost and st johns wart
o Alternative -acupuncture, reflexology, aromatherapy or homeopathy
o Psych - CBT - elevate low mood

How well did you know this?
1
Not at all
2
3
4
5
Perfectly
6
Q

Methods of administration, risks and benefits of HRT

A

Tablets; cream/pessary/ring (for vaginal dryness) ; transdermal e.g. patch/gel (dependent on skin conditions); implants (oestrogen/IUS)

Benefits

  • Improves vaginal dryness and sexual function, reduces vaginal atrophy
  • ↓ CV risk
  • Improved QOL - Improves symptoms, sleep, mood, BMD, etc
  • ↓ flushing frequency + severity
  • ↓ CRC risk – combined

Risks

  • ↑ Risk VTE (Stop 4 weeks pre-elective surgery)
  • ↑ Risk ischaemic stroke (>60 y/o) – oral HRT
  • ↑ Breast ca risk
    (combined) ; after stopping HRT risk returns to normal. Encourage breast awareness.
  • Unopposed - ↑ Endometrial and ovarian Ca risk
How well did you know this?
1
Not at all
2
3
4
5
Perfectly
7
Q

CI and SE of HRT

A

CI

  • Oestrogen-dependent Ca
  • Past PE
  • Undiagnosed PV bleeding
  • LFT ↑
  • Pregnancy
  • Breastfeeding
  • Phlebitis

SE

  • Weight ↑
  • ‘premenstrual’ syndrome
  • cholestasis;
  • vomiting
  • muscle cramps
  • irregular bleeding
How well did you know this?
1
Not at all
2
3
4
5
Perfectly
8
Q

What is an early menopause and what are the risk factors

A

Menopause - 40-45 y/o

Risk factors –> High BMI, FH, Early menarche, Low parity, hx oral contraceptive treatment

How well did you know this?
1
Not at all
2
3
4
5
Perfectly
9
Q

Ix and Mx of early menopause

A

Ix - FSH - > 40 Iµ/L x 2 (4-6 weeks apart)

Mx - HRT (may require higher oestrogen dose)

How well did you know this?
1
Not at all
2
3
4
5
Perfectly
10
Q

What is premature ovarian insufficiency and the cause.

A

Menopausal sx in women <40y/o - May be reversible

Cause - ↓ no. follicles at birth/ accelerated follicle atresia/ follicle or FSH R dysfunction

How well did you know this?
1
Not at all
2
3
4
5
Perfectly
11
Q

Sx, Ix, Mx and complications of POI

A

Sx - Amenorrhoea, ↑ gonadotrophins, ↓ Oestrogen (Hot flushes + vasomotor)

Ix - FSH x 2 raised (4-weeks apart) are diagnostic; Estradiol - low (<50 pmol/L)
o ? DEXA - Assess BMD and Tests for cause: Genetic karyotyping + Adrenal antibodies

Mx - MDT important to consider physical + psychological issues
o	Adcal D3 to prevent Osteoporosis and monitor CVD
o	HRT (cont/ cyclic) --> until age of natural menopause (~51 years old) with higher oest levels

Complications –> 80% greater chance of CV mortality, osteoporosis, Dementia

How well did you know this?
1
Not at all
2
3
4
5
Perfectly
12
Q

What are fibroids and describe the different types

A

benign growths of SM in the uterine myometrium

o Intramural (commonest) – confined to the myometrium of the uterus.

o Submucosal -immediately underneath the endometrium, protrudes into the uterine cavity.

o Subserosal - protrudes into and distort the serosal (outer) surface of the uterus. They may be pedunculated (on a stalk).

How well did you know this?
1
Not at all
2
3
4
5
Perfectly
13
Q

RF for fibroids

A
  • Obesity
  • Early menarche
  • ↑ age
  • FH
  • African-American
How well did you know this?
1
Not at all
2
3
4
5
Perfectly
14
Q

Presentation of Fibroids

A
  • Prolonged / HMB
  • Pressure sx (e.g. urinary) ± abdo distension
  • Chronic pelvic pain
  • Subfertility (? Obstructive fibroid)
  • Solid/ enlarged uterus – smooth, non-tender
How well did you know this?
1
Not at all
2
3
4
5
Perfectly
15
Q

Differentials of abnormal uterine bleeding

A
PALM - COEIN
PALM: Structural Causes
- Polyp (endometrial/cervical)
- Adenomyosis
- Leiomyoma (Fibroid)
- Malignancy &amp; hyperplasia

COEIN: Nonstructural Causes
Coagulopathy (esp vW)
Ovulatory dysfunction (↓ T3/4; PCOS; ↑ prolactin)
Endometriosis - <5%
Iatrogenic (IUD; TCA; warfarin; hormones)
Not yet classified (AV malformation)

How well did you know this?
1
Not at all
2
3
4
5
Perfectly
16
Q

Differential for enlarged uterus

A
Uterine
Fibroids
Pregnancy/ Molar pregnancy
Haematoma
Leiomyosarcoma/ endometrial carcinoma
Adenomyosis
Extra-uterine
Ovarian cyst
Ovarian malignancy
Ectopic pregnancy
Pyosalpinx
Hydrosalpinx
Primary fallopian tube neoplasm
Pelvic abscess
CRC
Bladder carcinoma
How well did you know this?
1
Not at all
2
3
4
5
Perfectly
17
Q

Investigations for enlarged uterus

A
  • Pregnancy test/ ßHCG levels
  • Bloods – FBC; iron studies, TFT; FSH+LH; oestrogen
  • Pelvic USS – Confirm fibroids diagnosis and size
  • ?MRI
How well did you know this?
1
Not at all
2
3
4
5
Perfectly
18
Q

Management of Fibroids

A

Medical - If Sx

  • NSAIDs / Tranexamic acid/ mefenamic acid – may reduce bleeding
  • Hormonal contraception –> COCP; POP; Mirena – control menorrhagia
  • GnRH agonists – ↓ fibroid size within a few months – use pre-op– use 6m only (risk osteoporosis). Often recur after stopping the medication.
  • Ulipristal acetate –>↓ size of fibroid and menorrhagia – use pre-op

Surgical

  • Myomectomy (excise fibrois S.E blood loss; adhesions; C-section)
  • Hysterectomy
  • Hysteroscopy and transcervical resection of fibroid (submucosal fibroids)

Radiological
UAE (Uterine artery embolisation) - occlude vascular supply to fibroid

How well did you know this?
1
Not at all
2
3
4
5
Perfectly
19
Q

Complications of fibroids

A
  • Recurrent miscarriage
  • Iron deficiency anaemia
  • Bladder / bowel symptoms
  • Torsion (penduculated fibroid)
  • Acute pelvic pain in pregnancy (red degeneration) - rapidly growing fibroid undergoes necrosis and haemorrhage

In pregnancy - Premature labour; PPH; Hydronephrosis; fetal malpresentation; IUGR

How well did you know this?
1
Not at all
2
3
4
5
Perfectly
20
Q

Pathophysiology of PCOS

A

Endocrine disorder –>excess androgen production and the presence of multiple immature follicles (“cysts”) within the ovaries.

↑ pulse frequency GnRH –> Excess LH –> ↑androgens –> testosterone)
o Suppress LH surge - no ovulation
o Follicles develop but arrested in early stage - ovarian cysts
o Unopposed oestrogen - ↑ risk endometrial hyperplasia
o Insulin resistance - ↑ levels insulin secretion –> suppress hepatic production SHBG –> higher levels of free circulating androgens.

How well did you know this?
1
Not at all
2
3
4
5
Perfectly
21
Q

RF and presentation of PCOS

A

Risk factors –> DM; Irregular menstruation; FH

Presentation

  • Amenorrhoea / Oligomenorrhoea
  • Weight gain/ obesity (+Insulin resistance
  • -> Acanthosis nigricans)
  • Masculinisation (Hirsutism, acne, male-pattern hair-loss)
  • Chronic pelvic pain
  • Depression (and other psychological symptoms)
  • Infertility
How well did you know this?
1
Not at all
2
3
4
5
Perfectly
22
Q

Differentials for PCOS

A
  • Hypothyroidisism
  • Hyperprolactinaemia
  • Cushing’s disease
  • CAH
How well did you know this?
1
Not at all
2
3
4
5
Perfectly
23
Q

Ix for PCOS

A

Rotterdam Criteria - 2 out of 3

1) Oligo- ± anovulation
2) Clinical ± biochemical signs of hyperandrogenism
- ↑ test + LH and ↓ SHBG +prog
- ↑ 3:1 LH:FSH
3) USS –> Polycystic ovaries - ≥ 12 peripheral ovarian follicles ± ovarian volume >10 cm3 (string of pearls)

Consider oral glucose tolerance test (esp if BMI >30).

How well did you know this?
1
Not at all
2
3
4
5
Perfectly
24
Q

Mx of PCOS

A

Treat underlying DM; HTN; dyslipidaemia and OSA

Oligomenorrhoea/Amenorrhoea
• Low dose COCP – control bleeding and ↓ risk unopposed oestrogen
• Dydrogesterone – a progesterone analogue –> withdrawal bleed (~3monthly)

Obesity -Aim BMI <30 - will help trigger regular menstrual cycle.

  • Healthy diet, exercise to ↑ insulin sensitivity, smoking cessation
  • In severe cases –> orlistat (pancreatic lipase inhibitor) can be prescribed.

Hirsutism

  • Cosmetically ± with anti-androgen medication (cyproterone, spironolactone or finasteride) - avoided during pregnancy as teratogenic.
  • Eflornithine (topical cream) - ↓ growth rate of facial hair.

Fertility
• 50mg Clomifene (Selective oestrogen R modulator) –>induce ovulation- 50-60% conceive in 6m
- ↑ risk multiple pregnancies, ovarian hyperstimulation syndrome, ovarian Ca (limit to 6 cycles)
• ± Metformin improves insulin sensitivity, helps menstrual disturbance and ovulatory function (recommended up BMI >25 and want to conceive)
• Women with a normal BMI could also benefit from laparoscopic ovarian drilling ( reduce test, and increase FSH - ovulate)

How well did you know this?
1
Not at all
2
3
4
5
Perfectly
25
Q

Complications of PCOS

A
  • Infertility
  • Miscarriage
  • Gestational DM/ T2DM
  • Endometrial hyperplasia –> malignancy
  • HTN/CVD
How well did you know this?
1
Not at all
2
3
4
5
Perfectly
26
Q

Classification and causes of amenorrhoea

A

Primary - failure to commence menses
Secondary - cessation of periods for >6m excluding pregnancy

Hypothalamic –> (↓ GnRH - ↓Fsh:LH )
- eating disorder; ↓/↑ T3/4 (affects LH/FSH); Kallman syn

Pituitary –> prolactinoma (I.GnRH sec); pitutary tumour; Sheehans syn (post-partum necrosis); AI/ radiation affecting pit; prolonged use contraception (downreg)

Ovarian - PCOS; Turners; POI; haemochromatomis (hypogonadism)

Adrenal gland - CAH

Genital tract abnormality - Ashermanns; Imperforate hymen; vaginal atresia; Androgen insensitivity; cryptomenorrhoea (hidden period as obstruction)

How well did you know this?
1
Not at all
2
3
4
5
Perfectly
27
Q

Causes and management of oligoemenorrhoea

A

Intervals between cycles >35d
o PCOS; Contraceptive/Hormonal treatments; Perimenopause
o Thyroid disease/Diabetes
o Eating disorders/excessive exercise
o Medications e.g. anti-psychotics, anti-epileptics

Mx

  • treat cause
  • COCP;POP
How well did you know this?
1
Not at all
2
3
4
5
Perfectly
28
Q

Pathophysiology and sx of dysmenorrhoea

A

As corpus luteum regress - ↓ progesterone  endometrial cells release prostaglandin

  • Spiral artery vasospasm - leading to ischaemic necrosis and shedding of the superficial layer of the endometrium.
  • ↑ myometrial contractions

Secondary - underlying pelvic pathology

Sx
Crampy lower abdominal pain, - starts at the onset of menstruation ± radiation to back/thigh. Lasts 48-72 hrs, worse at onset of menses.
Other Sx: malaise, N+V diarrhoea, dizziness. ± uterine tenderness

How well did you know this?
1
Not at all
2
3
4
5
Perfectly
29
Q

RF and differentials for dysmenorrhoea

A

RF:
Early menarche; Long menstrual phase;Smoking; Nuliparity; Heavy periods (clot dysmenorrhoea – pain when endometrium expels clot)

Differentials (diagnosis of exclusion)

  • Endometriosis (pain before period)
  • Adenomyosis
  • PID
  • Adhesions
  • IBS/IBD
How well did you know this?
1
Not at all
2
3
4
5
Perfectly
30
Q

Mx of dysmenorrhoea

A
  • Stop smoking; local application of heat
  • treat any underlying cause

1) NSAIDs (inhibit PGs) ± paracetamol
- Start few days before usual onset of pain
2) 3-6m trial of COCP/ POP/ Coil

How well did you know this?
1
Not at all
2
3
4
5
Perfectly
31
Q

Causes and Ix of heavy menstrual bleeding

A
  • 40-60% no underlying cause (Abnormal uterine bleeding)
  • Other: leukaemia, IUCD, PIC, hypothyroidism, Liver disease, SLE, Ca, Progesterone contraception, most commonly fibroids

Ix
• Bloods –> FBC (anaemia); TFT (?↓); hormones (?PCOS); Clotting screen (vW)
• USS – Detect Fibroids/ adenomyosis
• Urine pregnancy test (?ectopic)
• ?Hysteroscopy – biopsy pelvic masses
• ?Laparoscopy –abdo masses
• ?Endometrial biopsy – pathology/ inconclusive ± failure of pharm treatment

How well did you know this?
1
Not at all
2
3
4
5
Perfectly
32
Q

RF for heavy menstrual bleeding

A
  • Age (more likely at menarche and approaching the menopause) (>45 ↑ risk)
  • Obesity;
  • Previous caesarean section
How well did you know this?
1
Not at all
2
3
4
5
Perfectly
33
Q

Ix and Mx of Abnormal uterine bleeding

A

Ix - Smear up to date

  • STI screen
  • USS for structural pathology
  • ?laparoscopy (endometriosis)

1) Mirena Coil
2) tranexamic acid/NSAID or COCP/POP
3) Endometrial ablation/ hysterectomy

How well did you know this?
1
Not at all
2
3
4
5
Perfectly
34
Q

Ix for amenorrhoea/ oligomenorrhoea

A
  • Pregnancy test
  • TFT
  • Hormone levels (prolactin, GnRH, LH, FSH, Oest, test)
  • ?Karyotyping
  • USS
  • ? progest challenge (if get withdrawal bleed adequate oest but not ovulating)
How well did you know this?
1
Not at all
2
3
4
5
Perfectly
35
Q

Sx and mx of premenstrual syndrome

A
  • sx 5-11 d before period with >1 symptom free week after
  • sx –> bloating, breast tenderness, headaches, tension & irritability, fatigue, forgetfulness, mood swings, sleep problems, depression, carbohydrate craving, clumsiness, libido↓

Mx

  • support, lifestyle change, vit B6, COCP
  • Bromocriptine (Da Ag)
How well did you know this?
1
Not at all
2
3
4
5
Perfectly
36
Q

MOA and SE of Tranexamic acid

A

MOA - Antifibrinolytic - reversibly binds to lysine R on plasminogen/ preventing plasmin binding to and degrading fibrin.

SE - Headaches; Back ache; Abdominal pain

How well did you know this?
1
Not at all
2
3
4
5
Perfectly
37
Q

Example, MOA and SE of GnRH analogue

A

Zolidex (Nasal spray/ impant)

MOA - Analogue which activates GnRH R -↑ LH + FSH
Induces hypoestrogenic state –> amenorrhoea (due to pituitary desensitisation)

SE - Menopausal like symptoms e.g. night sweats, headaches, ↓ libido, mood swings

How well did you know this?
1
Not at all
2
3
4
5
Perfectly
38
Q

MOA; CI and SE of Ulipristal acetate

A

Used in fibroids and emergency contraception
- Selective Progesterone R modulator – acts by I. ovulation
CI – breast/gynae Ca/ hepatic disorder

SE
•	Dysmenorrhoea
•	Hepatic failure
•	Pelvic pain
•	Breast tenderness
How well did you know this?
1
Not at all
2
3
4
5
Perfectly
39
Q

MOA and SE of Danazol

A

Gonadotrophin inhibitior – suppressing the pituitary-ovarian axis by inhibiting the release of LH + FSH

SE - Androgenic effects
(hirsutism, acne); Breast atrophy; Hot flushes

How well did you know this?
1
Not at all
2
3
4
5
Perfectly
40
Q

Example, MOA, CI and SE of COCP

A

Microgynon –> Inhibits ovulation (stops LH surge), and reduces bleeding and period pain (Vaginal ring and patch work in same way)

CI - BMI>35/ breastfeeding; HTN; FH breast/cervical; hx migraines

SE
•	VTE
•	Breast / Cervical Ca
•	Headaches
•	Mood changes
How well did you know this?
1
Not at all
2
3
4
5
Perfectly
41
Q

Example, MOA, CI and SE of POP

A
  • Norethisterone
  • inhibit ovulation, thickening the cervical mucus
    SE
    Nausea; Headache; Bloating/ weight gain; Skin rash
How well did you know this?
1
Not at all
2
3
4
5
Perfectly
42
Q

MOA, CI and SE of Mirena Coil

A

Releases Levonorgestrel into the Uterus, preventing uterine shedding and reducing bleeding – thins endometrium and contraceptive

CI – recent STI/ hx PID/ endometritis

SE - Spotting; Ectopic pregnancy; Falling out

How well did you know this?
1
Not at all
2
3
4
5
Perfectly
43
Q

MOA and SE of MPA

A

Medroxyprogesterone acetate- Agonist of progesterone androgen and glucocorticoid R - i. follicular development and prevent ovulation
Thin endometrium + change cervical mucus

SE - menstrual disturbance/ acne; headaches; bone loss; blood clots

How well did you know this?
1
Not at all
2
3
4
5
Perfectly
44
Q

Types of Hysterectomy and SE

A

Subtotal (partial) - Uterus, not cervix
Total (Cervix + uterus)
+/- bilateral salpoingo-oophorectomy (fallopian tubes and ovaries)
Radical - Total + parametrium, vaginal cuff + part/ whole of fallopian tubes.

SE - Haemorrhage requiring blood transfusion

  • Early menopause (change in blood supply to ovaries)
  • Infection/ Pain/ Adhesions
  • Anaesthetic risks
  • total vault prolapse
Uncommon
Damage bladder/bowel
wound dehiscence
Pelvic abscess/ infection
Vaginal prolapse
VTE/ PE (early mobilisation)
How well did you know this?
1
Not at all
2
3
4
5
Perfectly
45
Q

UAE SE

A
  • Infection
  • Infertility
  • Failure
  • Pain and fever post - op
How well did you know this?
1
Not at all
2
3
4
5
Perfectly
46
Q

Types of endometrial ablation; SE and CI

A

Transcervical resection (GA) - uses diathermy
Balloon ablation (LA/GA) - using heated fluids
- need cervical dilation
Microwave energy (LA) - <72s
Bipolar mesh (LA/GA)
- need cervical dilation

SE
• Infection
• internal organ damage uterine perforation/ bowel/bladder
• Haemorrhage
• new adhesions
• intrauterine scarring - obstructed outflow of menstrual blood - haematometra + pelvic pain
• Fluid overload/ electrolyte imbalance ( transcervical)

CI
Only if don’t want any more children - risk placenta praecia +/- accerta
- Endometrial hyperplasia/ Ca

How well did you know this?
1
Not at all
2
3
4
5
Perfectly
47
Q

What is a vasectomy and the complications

A

LA; vas deferens cut/blocked

Complications - haematoma (scrotum); infection; epidymitis; sperm granuloma; testicular pain

How well did you know this?
1
Not at all
2
3
4
5
Perfectly
48
Q

Types of uterine prolapse

A

Anterior

  • Urethrocele (Lower) –> Prolapse of Urethra into the vagina; Stress incontinence
  • Cystocele (Upper) –> Prolapse of Bladder into vagina; Few / no symptoms; If large may cause increased frequency, recurrent UTI

Middle
- Uterine prolapse –> Descent of the Uterus into the vagina
1st ° –> cervix stays in the vagina.
2nd ° –> cervix protrudes from introitus when standing/ straining.
3rd ° –> the uterine fundus lies outside the vagina. The vagina becomes keratinized and the cervix may ulcerate (bleeding ± discharge from ulcer)
o Pressure, urinary incontinence, dyspareunia

  • Vaginal vault prolapse–> Descent of the vaginal vault after Hysterectomy; Often associated with cystocele, rectocele and enterocele’ Urinary retention, distal ureteric obstruction

Posterior
- Rectocele (Middle) - Prolapse of the rectum into the vagina (through levator ani); Sx - Constipation, urgency or symptomless - may have to reduce herniation prior to defecation
- Enterocele (Upper) –> Herniation of the Pouch of Douglas into the vagina
o Usually asymptomatic

How well did you know this?
1
Not at all
2
3
4
5
Perfectly
49
Q

RF for uterovaginal prolapse

A

Weakened pelvic floor:

  • Post-menopause (reduced oestrogen)
  • Pelvic surgery
  • Obesity / Frequent heavy lifting
  • Congenital/ genetics
  • Loss of muscle tone / Repeated straining (chronic constipation/COPD
  • Childbirth >1 –? Trauma / macrosomic fetus
How well did you know this?
1
Not at all
2
3
4
5
Perfectly
50
Q

Types of incontinence

A
Stress
Urge 
Mixed (stress and urge)
Overflow -
Functional
Fistulae
How well did you know this?
1
Not at all
2
3
4
5
Perfectly
51
Q

Pathophysiology and RF for stress incontinence

A

Weakness of pelvic floor and urethral sphincter leads to passage of urine when ↑ intra-abdo P
e.g coughing / laughing)

RF - weak pelvic floor (childbirth; obesity; chronic cough; post-pelvic surgery; post-menopausal

  • prostacetectomy ; TURP;
  • Infection; neurological disease; age; female
How well did you know this?
1
Not at all
2
3
4
5
Perfectly
52
Q

Pathophysiology and causes for urge incontinence

A

Detrusor overactivity causing high bladder pressure leading to sudden urge/ uncontrolable bladder emptying –> Large volumes, (day and night)

Causes

  • idiopathic (overactive bladder) - precipitated by arriving home; running water; coffee
  • Neurogenic (hyperreflexia): MS, PD, spinal cord injury (incl prolapsed disc/DM), stroke
  • Bladder outlet obstruction: prostate/ bladder/ cervical/ colon ca; BPH/stone/stricture; STI/trauma/ blood clot/ faecal impaction
  • UTI
How well did you know this?
1
Not at all
2
3
4
5
Perfectly
53
Q

Pathophysiology and causes for overflow incontinence

A

Bladder doesn’t empty properly/ incomplete voiding; overfill –> overflow
- hesitancy; terminal dribbling; poor stream

Causes

  • bladder outflow obstruction (e.g. BPH/stricture/stone/ faecal impaction)
  • impaired detrusor contractility (M.S/PD)
How well did you know this?
1
Not at all
2
3
4
5
Perfectly
54
Q

Reversible causes of incontinence

A
DIAPPERS
Delirium
Infection (UTI)
Atrophy
Pharm
Psych (demenita)
Excess UO (↑ intake/DI)
Restricted mobility
Stool impaction
How well did you know this?
1
Not at all
2
3
4
5
Perfectly
55
Q

RF for urinary incontinence

A

Predisposing
- Race; FH; Anatomy/neurology abnormality

Promoting
- Co-morbidites; ↑ intra-abdo P; cognitive impairment; UTI; pelvic prolapse; diet (caffeine, citrus fruit juice, alcohol), meds (diurectics; anti-cholinergics; anti-dep; sedatoves; alpha blockers

How well did you know this?
1
Not at all
2
3
4
5
Perfectly
56
Q

Red flags when present with urinary incontinence

A
  • haematuria
  • prolapse beyond introitus
  • pelvic mass
  • weight loss
  • pain assoc micturition
  • > 3 infections in 6m
How well did you know this?
1
Not at all
2
3
4
5
Perfectly
57
Q

Examination for urinary incontinence

A
  • general –> BMI (heigh/weight); MMSE
  • CVS - signs CR disease
  • CNS - gait; check L1-S3 sensation and neuro exam
  • Abdo - surgical scars, hernias, masses, distended bladder after voiding
  • DRE - impaction; tone; mass (prostate in males)
  • Vaginal - atophy; prolapse; urethral tenderness; strength pelvic floor (oxford scale) +/- cough/strain
  • speculum
How well did you know this?
1
Not at all
2
3
4
5
Perfectly
58
Q

Ix for urinary incontinence

A
  • freq/volume chart
  • 3 day bladder diary
  • urine dip +/- MC&S
  • Bloods - FBC (↑ WCC); U&E (↑ ca - constipated/confused); glucose (DM)

Imaging

  • USS kidneys- post-residual vol
  • cystoscopy (if haematuria)
  • USS abdo (CRF/obstruction)
  • CT abdo (mass)

Specialist - urodynamic studies

  • uroflowmetry
  • cystometry (get detrusor P)
  • video/ ambulatory urodynamics
How well did you know this?
1
Not at all
2
3
4
5
Perfectly
59
Q

Mx of uterovaginal prolapse

A

Conservative

  • watch and wait
  • lifestyle modification (stop smoking, constipation mx, weight loss)
  • pelvic floor exercises
  • vaginal pessary (frail) - S.E –> Vaginal discharge (smelly), discomfort, UTI, discomfort, ulceration , interfere with sex, incontinence
  • topical oestrogens (if post-meno

Surgery

  • hysterectomy
  • Sacrocolpopexy/ Sacrospinous fixation - to fix vaginal vault
  • colposuspension - strengthen pelvic floor (SE- Anaesthetic risk, infection, bleeding, DVT, voiding difficulty, urinary retention, urinary incontinence)
How well did you know this?
1
Not at all
2
3
4
5
Perfectly
60
Q

Mx of urinary incontinence

A

Stress

  • pelvic floor exercises (trial 3m)
  • vaginal tampon
  • duloxetine
  • vaginal tape (only if others fail - sling around urethra; SE - bladder perf; Damage to vessels/viscera; Voiding difficulties + urinary retention (short-term); Urgency and frequency; Groin/ suprapubic pain (short duration); Vaginal tape erosions.
  • Other –> rectus fascial sling; repair; injection of bulking agents into bladder neck

Urge
- bladder training (6wk) - pelvic floor+ scheduled voiding
- Reduced caffeine intake, weight loss, modify fluid intake (none after 8pm)
- Oxybutynin
Other –>botox injection; sacral N stim; self-catheter (if neuro)

Overflow - review meds/bowel/ intermittent catheter; education; rx cause

Functional - mobility aids; continence aids etc

How well did you know this?
1
Not at all
2
3
4
5
Perfectly
61
Q

MOA of duloxetine and SE

A

Inhibit pre-synaptic 5HT + NAdr in Onuf’s nucleus of sacral spinal cord
↑ [5HT+NAdr] in synaptic cleft Stronger urethral contractions and persistent sphincter tone during storage phase

SE
•	N&amp;V; GI Sx
•	Insomnia
•	Dizzy; drowsy
•	Anxiety; ↓ appetite
•	dry mouth; palpitation
•	flushing; headache
•	vision disorders (CI in glaucoma)
•	sweat changes
How well did you know this?
1
Not at all
2
3
4
5
Perfectly
62
Q

MOA of Oxybutynin and SE

A

Direct anti-spasmodic effect, acting on M1/2/3 SM R –> ↓ Detrusor activity - increase bladder capacity
Modified release tablets -↓ SE

SE
•	Dry mouth; headache
•	Urinary retention
•	Constipation/ diarrhoea
•	Blurred vision
•	Nausea; abdo pain
How well did you know this?
1
Not at all
2
3
4
5
Perfectly
63
Q

RF for PID

A
  • Sexually active/ Aged 15-24 y/o
  • Multiple sexual partners
  • Intercourse without barrier contraception
  • Hx of STIs/ PID
  • Instrumentation of cervix - gynae surgery, TOP, and insertion IUS
  • Retrograde menstruation
  • Hormonal changes – ↓ barrier from cervix (COCP ↓ risk)
How well did you know this?
1
Not at all
2
3
4
5
Perfectly
64
Q

What is PID

A

• Infective inflammation of endometrium, uterus, fallopian tubes, ovaries + peritoneum

  • Acute - <6 months; Chronic >6 months
  • ~ 25% - Chlamydia and gonorrhoea
How well did you know this?
1
Not at all
2
3
4
5
Perfectly
65
Q

Presentation of PID

A
  • asymptomatic
  • Lower abdo pain (unilateral / bilateral)
  • Deep dyspareunia/ Post-coital bleeding
  • Menstrual abnormalities (e.g menorrhagia, dysmenorrhoea or IMB)
  • Dysuria
  • Abnormal vaginal discharge (purulent ± unpleasant odour)

Advanced - severe lower abdo pain, fever (>38° C), and n&v

How well did you know this?
1
Not at all
2
3
4
5
Perfectly
66
Q

Ix for PID

A

Vaginal examination

  • tenderness of uterus/adnexae or cervical excitation
  • palpable mass in the lower abdomen
  • Speculum –> abnormal vaginal discharge noted ± cervicitis
  • Endocervical (gonorrhoea and chlamydia) + High vaginal swab (TV,BV)  NAAT
  • Full STI screen
  • Urine dipstick +/- MSU – to exclude UTI
  • Pregnancy test
  • ? FBC (CRP &WBC)
  • TV USS – severe/ diagnostic uncertainty.
  • ? Laparoscopy – severe
How well did you know this?
1
Not at all
2
3
4
5
Perfectly
67
Q

Complications of PID and when to admit

A
  • Infertility - (tubal adhesion)
  • Ectopic pregnancy - narrowing and scarring of the fallopian tubes
  • Fitz-Hugh-Curtis syndrome (peri-hepatitis) – infection of the Glissons capsule around the Liver, leading to acute RUQ pain and tenderness
  • Tubo-ovarian abscess (pocket of pus)
  • Chronic pelvic pain
  • Reiter syndrome

Admit –> pregnant; n&v/fever; signs abscess; need IV; HIV

How well did you know this?
1
Not at all
2
3
4
5
Perfectly
68
Q

Mx of PID

A

14-day Abx
• Low risk of Gonococcal –> Ofloxacin + Metronidazole BD
• High risk –> Ceftriaxone IM stat; then Doxycycline + Metronidazole BD

Advice
• Pain relief and rest
• Avoid sexual intercourse until Abx course complete and partner treated
• Contact tracing

How well did you know this?
1
Not at all
2
3
4
5
Perfectly
69
Q

RF for UTI

A
  • Sexual activity
  • New sexual partner
  • Pregnancy
  • Urinary/ faecal incontinence
  • Renal tract malformation • Use of spermicides (condoms)
  • ↓ oestrogen/menopause
  • Immunocompromised/ DM
  • Dehydration
  • Obstruction (stones/catheter)
How well did you know this?
1
Not at all
2
3
4
5
Perfectly
70
Q

Causes of chronic pelvic pain

A
Gynae
o	Endometriosis/ Adenomyosis
o	Chronic PID
o	Ovarian cysts
o	Post C-section
o	Prolpase
o	Fibroids
o	Tumours
GI
o	Adhesions
o	Appendicitis
o	Constipation
o	Diverticular disease
o	IBS

Urinary
o UTI
o Calculus

Other
o	MSK
o	Degenerative joint disease
o	Low back pain
o	Psychological 
o	CRPS
How well did you know this?
1
Not at all
2
3
4
5
Perfectly
71
Q

Pathophysiology and RF for endometriosis and adenomyosis

A

Endo - Chronic oestrogen-dependent condition –> growth of endometrial tissue in sites other than the uterine cavity (?retrograde menstruation), most commonly the pelvic cavity (incl. ovaries). Also includes uterosacral ligaments, the pouch of Douglas, peritoneum and lungs.

Adeno - extension of endometrial tissue and stroma into the uterine myometrium, affecting ~15% of women; causing a bulky, tender uterus and dysmenorrhoea

RF
• Early menarche / late menopause
• Delayed childbearing/ Long duration of menstrual bleeding
• Short menstrual cycles • FH – 1st ° relative
• HMB
• Defects of uterus or fallopian tubes

How well did you know this?
1
Not at all
2
3
4
5
Perfectly
72
Q

Presentation of endometriosis

A
  • Dysmenorrhoea (2°) /chronic pelvic pain
  • Menorrhagia
  • Dyspareunia (↑ in menses)
  • cyclical GI/urinary symptoms (?pain)
  • infertility - with 1+ of above

Present at distant sites - Haemothorax; epistaxis; rectal bleeding

Symptoms ↓ in pregnancy and menopause

How well did you know this?
1
Not at all
2
3
4
5
Perfectly
73
Q

Ix for endometriosis

A

Examination – can be normal
•fixed, retroverted uterus
•Enlargement (tender + boggy = adenomyosis)
•Tender nodularity in the posterior vaginal fornix and uterosacral ligament
•Visible vaginal endometriotic lesions or Adnexal mass
•Tenderness/ focal pain on examination

  • Pregnancy test – exclude ectopic pregnancy
  • Urinalysis + cervical swabs -exclude STI/PID cause of pain,
  • Laparoscopy(1st!) – visualise endometrial lesions
    >Chocolate cysts (active endometriosis); Adhesions (+scars - inactive); Peritoneal deposits
  • Pelvic USS –severity and demonstrate pelvic mobility ± bowel involvement
    MRI –visualise endometrial ectopic endometrial tissues
How well did you know this?
1
Not at all
2
3
4
5
Perfectly
74
Q

Mx of endometriosis

A

Medical

1) NSAIDs OR
2) COCP/vaginal ring - ↓ oestrogen
3) MPA OR Mirena Coil
4) Specialist
- GnRH agonist (e.g. soladex)

Surgical

1) Laparoscopic removal of ectopic endometrial tissue – Excision/ ablate
2) Hysterectomy with bilateral salpingo-oophorectomy (10% pain recurrence)
- replace hormones until menopause age
- Complications –> persistent pain; bladder/bowel injury; infection
3) Artificial insemination - ?IVF; potentially laparoscopic ablation

How well did you know this?
1
Not at all
2
3
4
5
Perfectly
75
Q

Presentation and RF for dyspareunia

A

RF - Peri / post-menopausal; Sexually inexperienced; hx sexual abuse

Presentation
•	Penetration / deep pain
•	Tightening of the vagina on penetration
•	Vulvodynia (vulva pain)
?Pelvic masses/ Suprapubic tenderness
How well did you know this?
1
Not at all
2
3
4
5
Perfectly
76
Q

Ix for dyspareunia

A
  • Examination – Assess for Vaginismus, and for tenderness on palpation
  • Cervical swabs – To assess for PID and STIs
  • Urinalysis – Assess for the possibility of an UTI cause
  • Laparoscopy – To assess for endometriosis
How well did you know this?
1
Not at all
2
3
4
5
Perfectly
77
Q

Mx of dyspareunia

A

oSex therapy - Modification of sexual technique + lubrication
oCBT
o Lidocaine – topically ( if pain alone the major complaint)

Treat underlying cause
e.g. vaginismus –> vaginal trainers and PT

How well did you know this?
1
Not at all
2
3
4
5
Perfectly
78
Q

What is an ectopic pregnancy and where can it happen

A

pregnancy which is implanted at a site outside of the uterine cavity

Tubal (>99%)
- Ampullary (55%)
- Isthmic (25%)
present late and often bleed catastrophically

Rare
Ovary; abdo; cervical

How well did you know this?
1
Not at all
2
3
4
5
Perfectly
79
Q

Rf for ectopic pregnancy

A

PMH

  • Previous ectopic
  • PID (adhesion formation)
  • Endometriosis (adhesions)
  • tubal pathology

Contraception

  • IUD/IUS
  • POP/ implant (fallopian tube ciliary dysmotility)

Iatrogenic

  • Pelvic surgery – especially tubal surgery
  • Assisted reproduction i.e. embryo transfer in IVF
How well did you know this?
1
Not at all
2
3
4
5
Perfectly
80
Q

Clinical features of ectopic pregnancy

A
  • Lower abdominal/pelvic pain ± irregular PV bleeding or hx of amernorrhoea
  • Vaginal discharge – brown (prune juice) –>result of decidual breakdown (as suboptimal β-HCG)
  • Shoulder tip pain (haemoperitoneum)

Other –> Breast tenderness, GI symptoms, dizziness, fainting or syncope, urinary symptoms, passage of tissue, rectal pressure or pain on defecation

How well did you know this?
1
Not at all
2
3
4
5
Perfectly
81
Q

Ix of ectopic pregnancy

A

On examination
- Abdo/ pelvic tenderness; enlarged uterus
Vaginal –>
- Cervical excitation +/- adnexal tenderness.
- Fullness in pouch of douglas (if ruptured)

If ruptured –> haemodynamically unstable
o pallor, ↑ CRT, tachycardia, hypotension, shock/collapse
o signs of peritonitis (abdo rebound tenderness and guarding)
o Abdominal distention

  • Pregnancy test
  • if +ve  pelvic USS (+/- TV USS) -
    If can’t see location
    (i) very early intrauterine pregnancy (ii) miscarriage (iii) ectopic pregnancy
  • serum β-HCG
    >1500 iU + no intrauterine –> ? ectopic –> diagnostic laparoscopy
    <1500 iU + pt stable –> blood test 48 hours later:
    • Viable pregnancy –> HCG level double every 48 hours
    • Miscarriage –> HCG level halve every 48 hours
How well did you know this?
1
Not at all
2
3
4
5
Perfectly
82
Q

Criteria for medical v surgical mx of ectopic pregnancy

A

Medical

  • No significant pain
  • Adnexal mass <35mm
  • No intra-uterine pregnancy on USS
  • Serum hCG <1500 IU/L
  • Not ruptured
  • Can attend for follow up (2 serum hCG day 4 &7; then every week until -ve)
  • 24h access to gynae services (+ sx rupture)

Surgical

  • Significant pain
  • Adnexal mass >35mm
  • Fetal heart beat visible on USS
  • Serum hCG >5000IU/L
  • Ruptured
  • Can’t attend for follow up
How well did you know this?
1
Not at all
2
3
4
5
Perfectly
83
Q

Medical v surgical mx of ectopic

A

Medical
+ve - Avoids surgical complications and the patient can be at home after the injection.
-ve - SE of methotrexate (abdo pain, myelosuppression, renal dysfunction, hepatitis, teratogenesis (use contraception for 3-6 m after use), treatment failure

Surgical
+ve: Reassurance about when the definitive treatment can be provided, high success rate.
-ve: GA risk, risk of damage to bladder/ bowel/ureters, DVT/PE, haemorrhage, infection, risk treatment failure (salpingotomy - some of the pregnancy may remain within the tube), ↑ risk of recurrent ectopic in salpingotomy, need Anti-D prophylaxis

How well did you know this?
1
Not at all
2
3
4
5
Perfectly
84
Q

Options for mx of ectopic

A

Conservative –> no sx/ rupture/ <3cm and hcg<1500
- sx of rupture and 24h access

Medical –> Methotrexate IM

Surgical –> Laproscopic salpingectomy

All - Miscarriage Association; Written information / leaflets; Counselling and Support

How well did you know this?
1
Not at all
2
3
4
5
Perfectly
85
Q

RF for miscarriage

A
•	 >35 (↑ ch. abnormalities)
•	Previous miscarriage (esp if ≥ 2 before)
•	Obesity 
•	Ch. abnormalities (maternal or paternal)
•	Smoking/ caffeine
•	Uterine anomalies e.g. fibroids
•	Previous uterine surgery
•	Anti-phospholipid syndrome/ SLE
•	Coagulopathies
•	PCOS
•	Poorly controlled DM/ thyroid
•	Serious maternal infection – causing fever --> Rubella, CMV can cross placenta
o	Malaria, chlamydia, listeria,syphilis
•	Alcohol (esp >5 units/wk) and drug use
•	Folate deficiency
•	Methotrexate
How well did you know this?
1
Not at all
2
3
4
5
Perfectly
86
Q

What is a miscarriage and sx

A

spontaneous loss of the fetus <24/40; occurs in 1/4 pregnancies

Sx

  • PV bleed +/- clots/POC
  • haemodynamically unstable
  • suprapubic pain
  • haemoperiteum
  • ? distended abdo
  • cervic open/closed
  • uterine tenderness/ adenxal mass/collection
How well did you know this?
1
Not at all
2
3
4
5
Perfectly
87
Q

Classifications and sx of miscarriage

A

Threatened - mild bleeding/pain; cervix closed (viable)

Inevitable - heavy bleeding/pain; cervix open (?viable)

Missed (silent) - on-going discharge; small for dates; asymptomatic but no fetal heart pulsation; anembryonic (empty gestation sac)

Incomplete - POC partially expelled (retained POC >15mm + proof pregnancy previously); cervix open

Complete - hx bleeding/clots/ POC; pain; cervix closed (previous proof and POC <15mm now)

Septic - Infected POC - fever; tender uterus; bleeding; pain

How well did you know this?
1
Not at all
2
3
4
5
Perfectly
88
Q

Mx of miscarriage

A

Threatened - observe

Inevitable - admit (if haemodynamiccal unstable)/ option for conservative/ med/ surgical mx

Missed - rescanned by 2nd person 7-14d to confirm-conservative/ med/ surgical mx

Incomplete - medical/surgical mx

Complete - discharge to GP

Septic - admit + sepsis 6 + remove infected tissue (med/surgically)

How well did you know this?
1
Not at all
2
3
4
5
Perfectly
89
Q

Differentials of miscarriage

A
  • Ectopic pregnancy
  • Hydatidiform mole
  • Cervical/uterine malignancy
How well did you know this?
1
Not at all
2
3
4
5
Perfectly
90
Q

Ix for miscarriage

A

+ve pregnancy test - send to EPAU

USS (?TV) –> fetal development + heartbeat (if >6/40)

  • yolk sac; ovarian cysts; free fluid/ mass; position of uterus
  • If no heartbeat 2nd scan 1-2 weeks later

Serum hCG
o> 1500 and no signs of pregnancy in uterus – significant
o<1500 – hCG 48 hours later ( if ↑ x 63% - intrauterine pregnancy; <63% - ?ectopic)

?FBC; blood group and Rh status; triple swaps and CRP (if pyrexic)

How well did you know this?
1
Not at all
2
3
4
5
Perfectly
91
Q

Mx of miscarriage

A

If >12 weeks ? Anti-D prophylaxis

-Conservative (<12/40) - wait
+ve: remain at home, no S.E. of meds/surgery, 24/7 access to gynae services
-ve: Unpredictable timing (takes longer), heavy bleeding + pain during passage of POC, may require further intervention + need for transfusion. Worries about being at home.

Follow-up: Repeat scan in 2/52 OR pregnancy test in 3/52.

  • medical (>14/40) –> Mifepristone + Misoprostol (48h later) - stimulate cervical ripening and myometrial contractions.
    + Analgesia + anti-emetics
    +ve: Can be at home, 24/7 access to gynae services, no anaesthetic/ surgical risk.
    -ve: S.E of meds: vom/d, heavy bleeding (up to 3/52) and pain during passage of POC, chance of requiring emergency surgical intervention.
    Follow-up: Pregnancy test 3 weeks later

Surgical - Surgical Evacuation of Retained products of Conception (SERPC) - using suction or manual vacuum aspiration with LA if <12 weeks
+ve: Planned procedure (help pt cope), unaware during the process (under GA), discharged same day
-ve: Anaesthetic risk, infection (endometritis), uterine perforation, haemorrhage, Ashermen’s syndrome, bowel/ bladder damage, retained POC, damage/weakness cervix
- histology on tissue

How well did you know this?
1
Not at all
2
3
4
5
Perfectly
92
Q

What is a recurrent miscarriage

A

Loss of ≥ 3 CONSECUTIVE pregnancies with SAME partner

How well did you know this?
1
Not at all
2
3
4
5
Perfectly
93
Q

Causes and mx of recurrent miscarriage

A
  • Balanced (Robertsonian) translocations
  • Uterine anomalies
  • Antiphospholipid syndrome
    o I. of trophoblastic function – poor oxygenation and nutrient supply to the foetus
    o Complement activation and inflammatory response at the maternal/foetal interface – Leading to poor oxygenation of the fetus - IUGR, Still birth, Pre-eclampsia
  • Thrombophilia –>hypercoagulability
  • Unexplained
How well did you know this?
1
Not at all
2
3
4
5
Perfectly
94
Q

Ix and mx for couples with recurrent miscarriage

A
  • anti-phospholipid Ab (2+ tests >12wk apart)
    o Mx - LMWH + lose-dose Aspirin + mx risk of DVT/PE; Stroke
  • Karyotype
  • Pelvic USS/ Hystereoscopy for uterine malformation +/- cervical weakness
  • thrombophilia screen
  • DM and hypothyroidism
How well did you know this?
1
Not at all
2
3
4
5
Perfectly
95
Q

What is GTD

A

Gestational trophoblastic disease –> Spectrum of disorders of trophoblastic development arising from abnormal ch no. in fertilisation (pregnancy related)

1) Pre-malignant conditions (more common) – such as partial(1 ovum and 2 sperm) /complete (empty egg and 1 sperm) molar pregnancy

2) Malignant conditions (rarer) - invasive mole/
choriocarcinoma (trophoblastic cells of placenta)

How well did you know this?
1
Not at all
2
3
4
5
Perfectly
96
Q

Staging of GTD

A
  • Stage 1 – Confined to Uterus
  • Stage 2 – Outside uterus, but limited to genital structures
  • Stage 3 – Extends to the lungs
  • Stage 4 – Affecting any other metastatic site
How well did you know this?
1
Not at all
2
3
4
5
Perfectly
97
Q

RF for GTD

A
  • <20 / >35
  • hx GTD/ miscarriage
  • Asian
  • oral contraceptive pill
How well did you know this?
1
Not at all
2
3
4
5
Perfectly
98
Q

Presentation of GTD

A
  • Vaginal bleeding + abdo pain in early pregnancy
  • Large uterus for dates (+soft/boggy)
  • Absence of fetal movement detected
  • Met signs (dyspnoea, etc)

Later
• Anaemia
• Hyperemesis (↑ β hCG)
• Hyperthyroidism (↑ β hCG stimulates thyroid)

How well did you know this?
1
Not at all
2
3
4
5
Perfectly
99
Q

Ix for GTD

A
  • Urine + blood hCG – remain ↑ after pregnancy
  • Histology POC – post treatment/ non-viable pregnancies
  • Pelvic USS – T2 onwards - complete mole =granular/ snowstorm appearance + central heterogeneous mass and surrounding multiple cystic areas/vesicles.
  • Staging CT CAP – if suspect mets
How well did you know this?
1
Not at all
2
3
4
5
Perfectly
100
Q

Mx of GTD

A

Register with GTD centre for follow-up and future monitoring

Surgical - Suction curettage –complete/partial molar pregnancy

Medical - evacuation (partial mole of a greater gestation with fetal development & isn’t conducive to surgical evacuation) – urinary β -hCG after 3/52

Chemotherapy – rising hCG after surgical evacuation, evidence of mets, or histological evidence of choriocarcinoma

Anti-D prophylaxis

After - 2/52 serum+urine hCG until normal (+ 4/52 if complete for 6m)

Advice - not to conceive until hCG levels have been normal for >6 months, and women who undergo chemotherapy are advised not to conceive for one year after completion of treatment

How well did you know this?
1
Not at all
2
3
4
5
Perfectly
101
Q

Presentation of Hyperemesis gravidarum

A

Prolonged and severe N&v of pregnancy (lasts >20/40) due to rapidly ↑ β – hCG levels
o >5% pre-pregnancy weight loss
o Severe dehydration
o Deranged bloods (e.g.↓ K/Na)
o Marked ketosis
o Nutritional deficiency/ muscle wasting (risk wernickes)

? mallory weiss tear; AKI; liver failure
- baby may have low birth weight/ be premature

Measure using PUQE score (Pregnancy-Unique Quantification of Emesis)

How well did you know this?
1
Not at all
2
3
4
5
Perfectly
102
Q

RF for Hyperemesis gravidarum

A
  • First pregnancy
  • Hx of hyperemesis gravidarum
  • Raised BMI
  • Multiple pregnancy
  • Hydatidiform mole
  • GTD
How well did you know this?
1
Not at all
2
3
4
5
Perfectly
103
Q

Differentials for Hyperemesis gravidarum

A
  • Gastroenteritis
  • Cholecystitis
  • Appendicitis
  • Hepatitis
  • Pancreatitis
  • Chronic H. Pylori infection
  • Peptic ulcers
  • UTI or pyelonephritis
  • Metabolic condition –> thyrotoxicosis, Addisons, DKA
  • Neurological conditions
  • Drug-induced (Abx / iron)
How well did you know this?
1
Not at all
2
3
4
5
Perfectly
104
Q

Ix for Hyperemesis gravidarum

A

Beside – Weight; Urine dip (ketones)

Lab
o MSU; FBC (anaemia, infection, haematocrit (can ↑)); Blood glucose (exclude DKA)
o U&E (↓ K/Na, dehydration, renal disease)

Severe causes
o LFT: exclude liver disease e.g. hepatitis or gallstones, monitor malnutrition
o Amylase: exclude pancreatitis
o TFT: ↑/↓-thyroid
o ABG: exclude metabolic disturbances, monitor severity

USS –>confirm viability, confirm gestation, exclude multiple pregnancy and trophoblastic disease/ ectopic

How well did you know this?
1
Not at all
2
3
4
5
Perfectly
105
Q

Mx for Hyperemesis gravidarum

A

Mild - Reassurance and rest +/- antiemetics (cyclizine) hydration, dietary advice

Moderate (or community mx failed) – ambulatory daycare.

  • IV fluids, parenteral antiemetics and thiamine. Managed until ketonuria resolves.
  • PPI for reflux, oesophagitis or gastritis

Severe – inpt mx

  • IV rehydration + KCL (guided by electrolytes)
  • IV antiemetics
  • Thromboprophylaxis

If can’t control may need termination

How well did you know this?
1
Not at all
2
3
4
5
Perfectly
106
Q

Choice of anti-emetics in Hyperemesis Gravidarum

A

1) Cyclizine; Promethazine; Chlorpromazine
2) Metoclopramide (max 5d due to risk EPSE); Domperidone; Ondansetron
3) Hydrocortisone IV (Once sx improve –> pred PO and wean)

How well did you know this?
1
Not at all
2
3
4
5
Perfectly
107
Q

Drugs that can can cause fetal abnormalities

A

In t1 - teratogenesis and T2/3 - affect growth

Li –> Ebstein anomaly, neonatal hypothyroidism, foetal hypotonia, poor reflexes, arrhythmia – strict monitoring

SSRI –> pulmonary HTN, (cardiac defects – paroxetine), miscarriage, low birth weight.
o Fluoxetine safer
oWithdrawal symptoms –> poor adaptation, jitteriness, irritability and poor gaze control

Valproate - ↑ risk NTD (use contraception)

Carbamazepine - cleft lip

Lamotrigine - ↑ risk of SJS

Anti-manic drugs - infants –>sedation, poor feeding, behavioural effects and developmental milestones.

Olanzapine - fetal macrosomia, GDM

How well did you know this?
1
Not at all
2
3
4
5
Perfectly
108
Q

Define Infertility

A
  • Primary –Couples who have inability to conceive >12m of regular unprotected intercourse
  • Secondary – Couples who have previously been able to become pregnant, but now cannot
How well did you know this?
1
Not at all
2
3
4
5
Perfectly
109
Q

Causes of infertility

A
  • Idiopathic
  • Female: ovulation disorder (PCOS, ovarian failure, hyperprolactinaemia); Anatomical anomalies (PID, STIs, sterilisation, fibroids, endometriosis); Turners
  • Male - disorders of sperm morphology/ vol/ concentration/ motility
How well did you know this?
1
Not at all
2
3
4
5
Perfectly
110
Q

Things to ask in the history regarding infertility

A

Female
Age; Methods of contraception
Duration + Type of infertility 1° /2°
-2° - previous birth e.g C-section/ PPH/ ectopic
Menstrual cycle – ovulating? Regular/irreg; Menorrhagia/ dysmenorrhoea/ amenorrhoea
Tubal surgery/ PID/ Pelvic surgery
pelvic pain – endometriosis/ fibroids
Taking folic acid; Smoking (↑ infertility)
Chemo/RT

Male
- Alcohol/ smoking (↓ sperm quality)
Previous surgery (urogenital) /infections (STI)
Sexual dysfunction – erectile/ ejaculatory
Varicocele; Cryptorchidism
Ask if had semen analysis (abnormal)
Chemo-RT

How well did you know this?
1
Not at all
2
3
4
5
Perfectly
111
Q

Ix for infertility in females

A

BMI

Pelvic exam -structural abnormalities; Fixed/ tender uterus

Mid-luteal progesterone - ? ovulating

Hormone levels (FSH + LH) (Day 2)
o ↑↑ may suggest poor ovulatory function
o High LH:FSH -?PCOS

Tubal patency assessment
o Hysterosalpingogram
o Diagnostic laparoscopy and dye

Genetic testing (Turners) and presence of 2 ° sexual characteristics

Additional: Pelvic USS (ovaries, uterine abnormalities); Hysteroscopy; Prolactin levels/TFTs; Testosterone/SHBG; Screen for chlamydia; Rubella status (?MMR vaccine. ? Booster)

How well did you know this?
1
Not at all
2
3
4
5
Perfectly
112
Q

Ix for infertility in males

A

Semen analysis x 2 from masturbation after >3d abstinence 3m apart

Hormone analysis (FSH + Testosterone)
o ↑ FSH may suggest impaired spermatogenesis

USS Testes
hx urinary sx / abnormal findings when exam testicles,

Genetic testing (Kleinfelters)

How well did you know this?
1
Not at all
2
3
4
5
Perfectly
113
Q

Normal semen analysis values

A
o Volume (ml) - >1.5ml
o Total sperm – 39 x 106 per ejaculation
o pH - >7.2
o Sperm concentration – 15 x 106 per ml
o Total motility ( % progressive &amp; non‑progressive motility): > 40% motile or > 32% progressive motility 
o Vitality: 58% or more live spermatozoa
o Morphology (normal forms) - >4%
How well did you know this?
1
Not at all
2
3
4
5
Perfectly
114
Q

Mx for female infertility

A

Dependent on cause
- unexplained –> IVF (>2 years not concieved)

  • tubal blockage –> Tubal anastomosis + reimplantation
  • Cervical factors - low pH (sodium bicarb douche); poor mucus quality (intra-uterine insemination)
  • Hypothalmic-pit failure –> lifestyles, BMI >19, pulsatile GnRH agonist (stim release FSH+LH - SE- hot flush; headache; osteoporosis)

HPG axis dysfunction (PCOS) - PCOS mx + clomifene citrate (block -ve feedback of oest - SE –>hot flushes, headche, n+v, ovarian enlargement) +/- laproscopic drilling + metformin

Ovarian failure –> IVF with donor egg

How well did you know this?
1
Not at all
2
3
4
5
Perfectly
115
Q

Mx for male infertility

A

Stop drinking/smoking; lose weight; loose fitting underwear and avoid sauna (reduce scrotal temp)

  • low semen vol –> IVF/ intra-uterine insemination
  • Oligospermia - clomifene citrate / IVF etc

Azoospermia - correct bloackage OR sperm extraction then injection

Hypogonadism - GnRH ag

Ejaculatory failure - sex therapy

How well did you know this?
1
Not at all
2
3
4
5
Perfectly
116
Q

Name some Assisted reproduction techniques

A

IVF - fertilise in a lab. Fertilised egg (zygote) is cultured for 2–6 d in growth medium and then transferred back to uterus to develop

Intra-uterine insemination - Placing sperm in uterus, to ↑ the no. of sperm that reach the fallopian tubes and subsequently ↑ the chance of fertilization

Intracytoplasmic sperm injection (ICSI) - single sperm is injected directly into an egg after being extracted from testicles

How well did you know this?
1
Not at all
2
3
4
5
Perfectly
117
Q

WHO principles of screening

A
  • Important health problem.
  • Should be a treatment for the condition.
  • Facilities for diagnosis and treatment should be available.
  • A latent stage of the disease.
  • Test/examination for the condition.
  • Test should be acceptable to population.
  • The natural history of the disease should be adequately understood.
  • Agreed policy on whom to treat.
  • Total cost of finding a case should be economically balanced in relation to medical expenditure as a whole.
How well did you know this?
1
Not at all
2
3
4
5
Perfectly
118
Q

Describe the cervical smear test process

A
  • 3-yearly from 25-49, and then 5-yearly to 50-64 year olds
  • smear at transformation zone –> detect CIN and HPV status

EITHER:

  • normal - routine recall
  • inadequate - repeat
  • abnormal borderline/ mild dyskaryosis but HPV -ve –> repeat smear 6-12m
  • Abnormal borderline/ mild dyskaryosis but HPV +ve –> Colposcopy
  • Mod/severe dyskaryosis –> urgent colposcopy +histology

Looking for abnormal vascular pattern (mosaicism, punctation); abnormal staining of the tissue (aceto-white, brown iodine) on colposcopy

How well did you know this?
1
Not at all
2
3
4
5
Perfectly
119
Q

What is CIN and the grades

A

Cervical intra-epithelial neoplasia (CIN) - abnormal growth of the cervical mucosa at transformation zone; it is potentially pre-malignant

CIN 1 (Mild) - Basal 1/3 of epi
CIN 2 (Mod) - Basal 2/3 of epi
CIN 3 (Severe) - >2/3 of epi /full thickness --> carcinoma in situ
How well did you know this?
1
Not at all
2
3
4
5
Perfectly
120
Q

RF for CIN

A
  • Most common aged 25-35 (though can occur at any age)
  • Young commencing sexual activity (<18)
  • Giving birth <16
  • Multiple sexual partners
  • Immunosuppression
  • Smoking
How well did you know this?
1
Not at all
2
3
4
5
Perfectly
121
Q

Mx if CIN

A

CIN1 –>ablation
CIN 2+3 –>
Excisional: LLETZ (large loop excision of the transformation zone) + histological analysis (under LA)
o Complications – Bleeding, infection, ↑ risk of premature rupture of membranes

Colposcopy after 6 m, then yearly for 10 yrs

Screen for STIs

Don’t treat while pregnancy

How well did you know this?
1
Not at all
2
3
4
5
Perfectly
122
Q

Describe the breast cancer screening process

A

47-73 every 3 years - mammogram (or <47 +RF; if <35 - USS)

RESULTS:
- normal
- abnormal –> breast exam; repeat mammograph +/- breast USS
of the abnormal results 1/4 have cancer

How well did you know this?
1
Not at all
2
3
4
5
Perfectly
123
Q

What is endometrial hyperplasia and what causes it

A

Precancerous prolif of
endometrium –> ↑ volume of endometrial tissue –> abnormal thickening of the lining of the uterus - ↑ risk endometrial ca (hyperplasia +/- atypia)

Cause–> Excess/ unopposed oestrogen
o early menarche, late menopause, PCOS –> ↑ anovulatory cycle
o Exogenous oestrogen therapy (e.g. HRT); Obesity; Low parity

How well did you know this?
1
Not at all
2
3
4
5
Perfectly
124
Q

Diagnosis and mx of endometrial hyperplasia

A

SX –> AUB

Ix –> TV USS - thickened endometrial stripe ↑ suspicion
-Confirm diagnosis - endometrial biopsy

Mx
- Hyperplasia-atypia
o Hormonal - Mirena and surveillance biopsy to identify progression or high dose progesterone

  • Atypical hyperplasia
    o Total hysterectomy ± bilateral salpingo-oophorectomy
    If CI - regular surveillance biopsies
How well did you know this?
1
Not at all
2
3
4
5
Perfectly
125
Q

Types +RF and presentation of endometrial cancer

A

1) Endometrial adenocarcinoma- related to unopposed oestrogen; Obesity; Lynch; endometrial hyperplasia and increasing age
2) Other: papillary serous; clear cell etc - oest indep

Presentation
AUB – intermenstrual, irregular menstruation, postmenopausal (10% risk)
- Less commonly blood stained, watery, purulent vaginal discharge

Abnormal cervical smear

Advanced/ mets (cervix, bladder, rectum, regional LN)
- Abdo pain/distention; weight loss; feeling of fullness in the abdo, abnormal bowel/ bladder function –> CT CAP

How well did you know this?
1
Not at all
2
3
4
5
Perfectly
126
Q

Ix for postmenopausal bleeding

A
  • Abdo exam– masses
  • Speculum - vulval/vaginal atrophy, or cervical lesions.
  • Bimanual– assess size & axis of uterus before endometrial sampling
  • TV USS – ix PMB- >4mm endometrial thickness need to pipelle biopsy for histology (of <4 and recurrent AUB)

Hysteroscopy with biopsy - AUB, multiple RF for Ca or v. thick endometrium on USS

How well did you know this?
1
Not at all
2
3
4
5
Perfectly
127
Q

Mx of Endometrial Ca

A
  • Weight loss
    Stage 1) Total hysterectomy + bilateral salpingo-oophorectomy + Peritoneal washings
    Stage 2) Radical hysterectomy + assess & remove pelvic LN ± adjuvant radiotherapy.
    Stage 3) Chemo +/- RT +/- max de-bulking surgery
    Stage 4) RT / high dose oral prog
How well did you know this?
1
Not at all
2
3
4
5
Perfectly
128
Q

Stages of endometrical Ca

A

Stage 0 – carcinoma in situ
Stage 1 - confined to organ of origin
Stage 2 – Invasion of surrounding organs/tissues
Stage 3 – Spread to distant nodes/ tissue in pelvis
Stage 4 – Distant mets

How well did you know this?
1
Not at all
2
3
4
5
Perfectly
129
Q

Differentials for Post-menopausal Bleeding

A
  • trauma/ bleeding from elsewhere
  • oestrogen-secreting ovarian tumours e.g. granulosa-thecal cell or oest from HRT
  • endometrial –> atrophy; polyps; hyperplasia; cancer; endometritis/PID
  • Cervical –> polyps; cancer; cervitis
  • vaginal – atrophic vaginitis, vaginal cancer
  • vulval pathology – dermatitis; dystrophy; cancer
  • systemic problem – bleeding disorders, mets cancer e.g. ovarian, CRC
How well did you know this?
1
Not at all
2
3
4
5
Perfectly
130
Q

Pathophysiology and RF for cervical cancer

A

Mainly SCC (99% caused by HPV) - developed from CIN

Other - adenocarcinoma/ mixed

RF - • HPV infection (16 + 18)
•	Multiple sexual partners
•	Young age 1st intercourse
•	Exposure (no barrier contraception)/ other STIs	
•	Immunosuppression/ HIV
•	Smoking
•	 use COCP >8 years)
•	Non-compliance cervical screening
How well did you know this?
1
Not at all
2
3
4
5
Perfectly
131
Q

Presentation of cervical cancer

A
  • Screening
  • AUB (+ can see ulcer on speculum)
  • Vaginal discharge (blood-stained, foul-smelling)
  • Dyspareunia
  • Pelvic pain
  • Weight loss

Late symptoms

  • Painless haematuria, painless PR bleeding, leg oedema, altered bowel habit, loin pain, radiculopathy
  • ?pelvic masses.
  • GI–hydronephrosis, hepatomegaly, rectal bleeding, mass on PR.
How well did you know this?
1
Not at all
2
3
4
5
Perfectly
132
Q

Ix for cervical cancer

A
  • test for chlamydia if <50 y/o (if +ve treat)
  • colposcopy + biopsy

Confirmed - baseline bloods and CT CAP +/- MRI pelvis

How well did you know this?
1
Not at all
2
3
4
5
Perfectly
133
Q

Mx of cervical cancer

A

1a) Radical trachelectomy (preserve fertility) - removal of the cervix and upper vagina OR
laparoscopic hysterectomy
1b) Radical hysterectomy +/- RT+ neoadjuvant/ adjuvant chemo
2) Radical hysterectomy +chemoradiation
3) Chemoradiation
4) Surgery/ pallative chemo

Review every 4m after treatment complete for 2 years then 6m 3 years

How well did you know this?
1
Not at all
2
3
4
5
Perfectly
134
Q

Stages of cervical ca

A

Stage IA: Invasive cancer identified only microscopically.

Stage IB: Gross lesions confined to the cervix or preclinical lesions greater than

Stage II: extends beyond the cervix, but does not extend into the pelvic wall. The carcinoma involves the vagina, but not as far as the lower third.

Stage III extended into the pelvic sidewall/ lower third of the vagina/ hydronephrosis

Stage IV extended to involve the mucosa of the bladder and/or rectum/ mets

How well did you know this?
1
Not at all
2
3
4
5
Perfectly
135
Q

Complications of surgery and RT for endometrical ca

A
--> Surgery 
Infection
VTE
Haemorrhage
Vesicovaginal fistula
Bladder dysfunction
Lymphocyst formation
Short vagina
--> RT
Vaginal dryness
Vaginal stenosis
Radiation cystitis
Radiation proctitis
Loss of ovarian function
How well did you know this?
1
Not at all
2
3
4
5
Perfectly
136
Q

RF and protective factors for ovarian cancer

A
RF
•	Genetics (BRCA1/2 or Lynch 2 syn)
•	Age (>50)
•	Smoking
•	FH 	
•	Extended oestrogen exposure (early menarche, late menopause, nulliparity, Oest only HRT, Obesity)
•	Endometriosis
Protective
•	Multiparity/ pregnancy
•	Breast-feeding
•	Sterilisation	
•	Early menopause
•	COCP
•	Hysterectomy/ oopherectomy
137
Q

Presentation of ovarian ca

A
~60% present late (stage 3-4)
• Insidious onset vague sx
• Abdo discomfort / bloating
• Urinary frequency / PV bleed
• Ascites
• Change in bowel habit/ IBS
• Dyspareunia	
• Constitutional symptoms (fatigue, weight loss, cachexia)
• Pelvic / back pain
o Acute -->bleeding into the cyst, rupture or torsion (?shock)
o Chronic --> due to pressure
o Cyclic -->endometriosis
138
Q

Ix for Ovarian Ca

A

• Abdo exam –> pelvic masses/ ascites
• Pelvic examination –» discharge/ bleeding, adnexal masses and cervical excitation.
• Bloods – FBC (chronic bleed, anaemia), U+E, LFT, and albumin
oCA125 – Ovarian Ca marker (any peritoneal irritation will raise it)
• Abdo and pelvic USS – To assess for masses–> risk of malignancy index

Confirmed Ca - Staging CT CAP and CXR

139
Q

Mx of ovarian ca

A

Counselling (if childbearing age)
- staging laparotomy (↑ RMI) + attempt to debulk the tumour.

  • Oophorectomy (repro age) OR Total hysterectomy with bilateral salpingo-oophrectomy

Adjuvant chemotherapy – all but early, low grade

F/u –clinical exam+ monitoring of CA125 level for 5 years

Recurrence – palliative chemotherapy

140
Q

Staging of ovarian Ca

A

o Stage 1 – Limited to ovaries
o Stage 2 – Involving one/both ovaries with pelvic extension
o Stage 3 – Involving one/both ovaries with microscopic peritoneal implants
o Stage 4 – With distant metastases

141
Q

Red flags for ovarian Ca

A
- >50y/o who present with:
> Abdo mass/ ascite
OR
o	Abdominal distension ('bloating')
o	Early satiety ± loss of appetite
o	Pelvic / abdominal pain
142
Q

Classification of ovarian Ca

A

Epithelial 90% –> >50 years
- Serous/ mucinous cystadenocarcinoma

Germ cell –> <35 y/o
- presents early; may sec hcg/ AFPO

Mets

143
Q

Criteria used to identify risk of malignancy in ovarian ca

A

Risk of malignancy index RMI = U x M x CA125

Menopausal status (M)		Pre- menopausal (1) Post- 
menopausal (3)

Ultrasound score (U) –> 1 feature = 1; >2 = 3

  • Mulilocular cyst
  • Solid areas
  • Metastases
  • Ascites
  • Bilateral lesions

CA125–> units/ml

144
Q

Types of Ovarian cyst

A

Functional
- follicular and corpus luteal

Pathological

  • endometrioma (chocolate cyst)
  • PCOS
  • Theca lutein cyst (molar pregnancy)

Benign

  • epi –> serous/ mucinous cystadenoma
  • germ cell –> mature cystic teratoma (+teeth/hait/skin/bone)
  • sex-cord –> fibroma/ theca cell
145
Q

Mx of Ovarian cyst

A

Pre-menopausal - rescan in 6 weeks - if still there–> then monitor with USS + Ca125 6m
- consider cystecomy

Postmenopausal - Use RMI
• RMI <25 -follow up for 1 yr with USS and CA125 if less than 5cm.
• RMI 25-250- bilateral oophorectomy; if Ca - staging + completion surgery of hysterectomy, omentectomy ±lymphadenectomy).
• RMI >250 - referral for staging laparotomy

146
Q

Pathophysiology and RF for vulval cancer

A

85% squamous - majority on labium majorium

RF
•	VIN
•	HPV infection	
•	Lichen sclerosis/ sw cell hyperplasia
•	Pagets disease of the vulva
147
Q

Presentation of vulval cancer

A
  • Vulval lump
  • Vulval bleeding / ulceration
  • Vulval pruritus /pain

Affecting labia majora – keratotic, warty and ulcerated lesions

148
Q

Ix + Mx for vulval Ca

A

Exam– extent + borders of lesion - Anaplastic lesions more likely to invade deeply

Biopsy – To provide a histological diagnosis

CT CAP – For staging (often metastasise via inguinal LN)

Mx

  • counselling (surgery may mutilate - need reconstruction)
  • chemo/ RT
  • radical vulvectomy +/- remove LN
149
Q

What is VIN

A

Vulval intra-epithelial neoplasia (VIN) - • A pre-malignant state of epithelium of the Vulva, no screening tests - usually related to HPV

Sx - asymp or itchy/burn/pain

150
Q

Mx of atrophic vaginitis of endometrial strophy

A

• HRT / Topical oestrogen cream
Works by thickening and re-vascularising the vaginal epithelium, so improving lubrication; and reducing fissure development

151
Q

How is a pregnancy dated

A

Naegele rule

1st day LMP + 1year -3m +7d

152
Q

How do you assess gestational age and what should the assessment show

A

USS between 10 weeks and 13 weeks 6 days

  • gestational age using CRL (10 Weeks = 3cm; 12 weeks = 5cm; 13 weeks = 7cm)
  • fetal viability (looking at cord; skull and abdo wall)
  • multiple pregnancies
153
Q

Early presentation of pregnancy

A

Sx
- Amenorrhoea - endometrium maintained with prog from
Corpus luteum
- ↑ frequency of micturition, especially at night
- Nausea + vomiting
Mx - ginger, wrist acupressure, antihistamines
- Breast - tingling of the nipples and areola, >6 weeks they enlarge

Signs
- Breast changes
o ↑ in size and feel warm
o Areolae darken
o Montgomery’s tubercles become prominent (sebaceous glands on the nipples)
- Cervix changes –>Appears lilac red on passing a speculum
- Uterus –>Enlarges as it accommodates the fetus

154
Q

Symptoms later on in pregnancy

A

Heartburn; constipation; haemorrhoids; vaginal discharge; varicose veins (give support stockings); backache (massage, exercise in water, back care classes)

155
Q

Physiological changes occuring in pregnancy

A

Resp - ↑ tidal volume; ↓ total lung capacity; ↑ oxygen consumption; ↓ pCO2

CVS - CO ↑; ↑ blood volume

Urinary - ↑ GFR –>↓ plasma urea + cr/ glycosuria; ↑ urinary frequency; ↑ renal length (cellular hypertrophy) + ↑ ureter diameter

Blood -↑ WBC; ↑ platelets +CF (hypercoaguble state); ↓ Hb;

Metabolic - HPL –> ↑ blood sugar and insulin resistance; weight gain (~12.5kg - Fetus, uterus, breasts, ↑ blood vol and body fat)

GIT - SM relax (prog) –> ↑ risk haemorrhoids + pain; GOR; constipation (↓ motility)
+ Displaced organs upwards

Water - ↑ - softens ligaments +/- CTS

Endocrine - hyperthryoid (stim hcg)

156
Q

Things to discuss at the first antenatal appointment

A

• smoking cessation, alcohol (non 3m than 1-2 unit/week), diet, recreational drugs

folic acid supplementation + Vit D supplements

Food hygiene and ↓ risk of a food-acquired infection (listeriosis and salmonella)
o Drink pasteurised/ UHT milk
o NOT EATING -ripened soft cheese (Camembert, Brie); pâté; uncooked/ undercooked ready‑prepared meals; raw/partially cooked eggs or food that may contain them (eg. mayo); raw or partially cooked meat, esp poultry

moderate exercise (+ pelvic floor exercises)

Screening: anaemia, HBV, HIV, rubella status, syphilis, chlamydia, SCD/ thalassaemia; ABO, rhesus status and any irregular RBC Ab

Downs screen - Nuchal thickening can be performed until 14 weeks

Info- how the baby develops; place of birth & pregnancy care pathway; breastfeeding (offer workshops); ? antenatal classes; Maternity rights and benefits; Majority are safe to continue work; Air travel - ↑ risk VTE; Mental health issues

RISK ASSESSMENT + risks for pre-eclampsia, gestational diabetes etc

157
Q

What is looked for at the 20 weeks scan

A

Structural anomalies

  • all limbs
  • encephaly
  • cleft lip
  • scoliosis
  • spina bifida
  • heart working
  • stomach
  • kidneys

+/- gender
+ confirm due date

158
Q

When is anti-D prophylaxis offered

A

Week 28 and week 34

or <72h of a sensitising event

159
Q

What are the RF, complications and presentation of a pregnancy >42/40

A

RF - Nulliparity; >40 y/o; Previous/ FH hx; ↑ BMI; unaccurrate dating

Complications

  • ↑ risk still birth - ↑ potential for placental insufficiency
  • ↑ risk of fetal acidaemia + meconium aspiration in labour
  • ↑ need for instrumental/ C-section delivery
  • ↓ oxygen and nutrient transfer - placental degradation can deplete fetal glycogen stores, resulting in neonatal hypoglycaemia

Presentation

  • macrosomia/ oligohydramnios/ reduced fetal movements
  • meconium
  • dry/ flaky skin (less vernix|)
160
Q

What are Braxton-Hicks contractions

A

From T1 - uterus undergoes physiological contractions (‘practice contractions’)

Common after week 36

+/- pain; occur infrequently; Cease with activity (usually); Very short duration (~30 seconds); Always have the same intensity (contractions proper increase intensity)

161
Q

Factors affecting labour

A

Passage (bony pelvis)

Power (of contractions - progressive shortening)

Passenger (moulding of cranium)

162
Q

Stages of normal labour

A

Expulsion of cervical mucus plus
1) dilation –> usually 1-3cm/hr (quicker in parous)- from onset of regular contractions to full dilation of cervix.

2) complete dilation –> baby born. Urge to push + valsalva maneouvre (engage - flex - descend - extend - external rotation)
3) delivery of placenta and membranes - give oxytocin (promote uterine contraction) to reduce to <5 mins - ↓PPH

163
Q

How do you monitor labour

A

VE - 4 hrly –> dilation +position

Maternal urine - 4 hrly –> ketones + protein - (+ve ketones - 10% dextrose IV)

Baseline Obs - 30 mins - ?distress

CTG –> fetal heart and uterine contractions

LAPPED fetus position
L – Lie
A – Attitude (posture, flexed / extended head)
P- Presentation
P – Position (part of head that exits birth canal)
E – Engaging diameter
D – Denominator (the presenting part, eg. Occiput, face, brow, etc)

164
Q

Causes and mx of delayed first stage of labour

A
  • membranes not ruptured
  • 1st baby
  • inadequate uterine activity
    Mx - Start syntocin; DVE for process; if indicated emergency C-section
165
Q

Interpreting CTGs

A

DR C BraVADO

Define Risk (pre-eclampso; DM; preterm)

Contractions - aim 4/5 in 10min); should be <1min and intensity should increase

Baseline rate - Fetal HR 110-160 (every 15m 1st stage + 5m 2nd)

Variability - can fluctate by 5-25bpm; FBS if <5bpm for >90mins

Accelerations - >15bpm for 15sec

Deceleration (<15bopm for 15s) - AFTER contraction sign of fetal hypoxia - FBS; ANYTIME - umbilical cord compression

Overall impression - suspicious/ pathological –> senior help

166
Q

What circumstances may require you to attach a CTG

A

Meconium ileus; abnormal fetal HR; mother pyrexial; oxytocin/ epidural use; PV bleed; PROM; APH; multiple pregnancies; DM/HTN/PRe-eclampsia; IOL; prolonged pregnancy

167
Q

Causes of fetal tachycardia

A

Hypoxia
Hyperthyroidism
Fetal / maternal anaemia
Chorioamnionitis (if maternal fever present)

168
Q

Causes of fetal bradycardia +/- prolonged

A
  • Post-date gestation
  • Occiput posterior / Transverse presentation

Prolonged (<80bpm for >3m)

  • Prolonged cord compression
  • Cord prolapse
  • Rapid foetal descent
169
Q

Common post-natal problems

A

o perineal pain, discomfort, stinging, offensive odour or dyspareunia (use topical cold therapy + paracetamol)

o Headache - ?post epidural/ spinal; ?tension/ migraine headache

o Persistent fatigue –?FBC

oConstipation – assess diet and fluid intake ± gentle laxative

o Haemorrhoids – if severe/ prolapsed – urgent action.

o Faecal incontinence – assess severity, duration and frequency

o Urinary retention (< 6 hrs of birth) – advise warm bath/shower when urinarate; ?catheter

o Urinary incontinence – pelvic floor exercises

o Mental health problems – ensure coping strategies and support in place and encourage to report concerns

170
Q

Hormones that affect lactation

A

↑ progesterone & oestrogen inhibit lactation before birth (↓ after)

Prolactin - maintaining tight junctions of the ductal epithelium and regulating milk production

Oxytocin - contracts SM in alveoli - squeeze milk into duct system

171
Q

What is in the first milk the baby recieves

A

colostrum - ↑ WBCs + IgA, which coats the lining of the baby’s immature intestines - helps to prevent pathogens from invading

Over the first 2 weeks after birth, colostrum production slowly gives way to mature milk

172
Q

Benefits of breastfeeding

A

To baby
- ↓ infection; childhood leukaemia; cot death; T2DM
To mother
- ↓ breast/ovarian Ca; postnatal depression; osteoporosis; T2DM/ Obesity§

173
Q

What effect does obesity have on pregnancy

A

↑ risk - difficulty conceiving, miscarriage, gestational DM, caesarean, HTN/ pre-eclampsia, VTE, anaesthetic complications and wound infections

Babies of obese mothers are ↑ risk of – stillbirth, NTD, prematurity, macrosomia, shoulder dystocia, neonatal death and obesity.

174
Q

What effect does smoking and alcohol/drugs have on pregnancy

A

Smoking - : IUGR, miscarriage/stillbirth, prematurity

Alcohol - fetal alcohol syndrome - growth restriction, LD, facial anomalies

Cocaine - spontaneous abortion, placental abruption, premature birth, IUGR and sudden infant death syndrome

175
Q

What are the risks of pregnancy in older women

A

↑ global risks: difficulty in conceiving, ↑ risk of miscarriage, twins, fibroids, hypertension, gestational DM, labour problems and perinatal mortality with ↑ maternal age

↑ Downs syn risk -
1 in 800 at 30 yrs
1 in 100 at 40 yrs
1 in 50 or more at >45 yrs

176
Q

How is amniotic fluid produced

A

Comprised of fetal urine output (main); placenta and some fetal secretions (e.g. respiratory).

The fetus breathes and swallows the amniotic fluid. It gets processed, fills the bladder and is voided, and the cycle repeats.

177
Q

Define and name the causes of oligohydramnios

A

amniotic fluid index < 5th centile for GA

Causes

  • PPROM
  • Placental insufficiency –blood flow redistributed to fetal brain rather than the abdo + kidneys - causes poor urine output.
  • Renal agenesis (Potter’s syndrome)
  • Non-functioning fetal kidneys, e.g. bilateral multicystic dysplastic kidneys
  • Obstructive uropathy
  • Genetic/chromosomal anomalies
  • Viral infections
178
Q

Define and name the causes of polyhydramnios

A

amniotic fluid index > 95th centile

Causes

  • Idiopathic in 50-60% o
  • fetus swallowing stopped – e.g. oesophageal atresia, CNS abnormalities, muscular dystrophies, congenital diaphragmatic hernia obstructing oesophagus
  • Duodenal atresia – ‘double bubble’ sign on USS
  • Anaemia – alloimmune disorders, viral infections (TORCH)
  • Fetal hydrops
  • Twin-to-twin transfusion syndrome
  • ↑ lung secretions – cystic adenomatoid malformation of lung
  • Genetic/ ch abnormalities
  • Maternal DM – esp if poorly controlled
  • Maternal ingestion of Li – leads to fetal DI
  • Macrosomia – LGA produce more urine.
179
Q

What is TORCH infections

A
Group of infections that cause proboems for developing baby 
Toxoplasmosis
Other (coxsackie, syphilis chicken pox)
Rubella
CMV
HSV
180
Q

Ix of abnormal amniotic fluid

A

Hx- sx of leaking fluid and feeling damp all the time

Examination

  • S-F height; tense uterus? (poly)
  • ?PROM; maternal DM (GTT)

Ultrasound

  • ↑/ ↓ amniotic fluid
  • Assess for liquor volume
  • Measure fetal size (SGA from placental insufficiency)
  • Fetal anatomy – structural causes, renal agenesis and obstructive uropathy
  • ?↑ in pulsatility index of the umbilical A Doppler in placental insufficiency.

Karyotyping (if appropriate) – particularly in cases of early and unexplained/ structural abnormality
Doppler – fetal anaemia (poly) OR Assses kidneys (oligo)
TORCH screen

181
Q

What is PROM/ PPROM

A

Premature rupture of membranes (PROM) –rupture >1 hr prior to the onset of labour, at ≥37/40

Pre-term premature rupture of membranes (P-PROM) – rupture of fetal membranes - <37/40

182
Q

RF that may lead to early membrane rupture

A
  • Smoking (esp < 28/40).
  • Previous PROM/ pre-term delivery (predisposition)
  • Vaginal bleeding during pregnancy.
  • Lower genital tract infection (weaken fetal membrane)
  • Invasive procedures e.g. amniocentesis (activate normal process early)
  • Polyhydramnios.
  • Multiple pregnancy.
  • Cervical insufficiency
183
Q

Clinical features of PROM

A
  • Gradual leakage of watery fluid from vagina + damp underwear/pad
  • Speculum  Don’t do if SEE fluid draining
    opooling in the posterior vaginal fornix. Lay on couch for at least 30 mins.
    o Coughing during examination – amniotic fluid expelled
  • Change in colour/ consistency or lack of normal vaginal discharge (‘washed clean’)
  • Avoid DVE until in active labour - ↓ time between rupture & onset of labour (↑ risk asc intrauterine infection)
184
Q

Differentials of PROM

A
o	Urinary incontinence.
o	Normal vaginal secretions of pregnancy.
o	↑ sweat/ moisture around perineum.
o	↑ cervical discharge (e.g. infection).
o	Vesicovaginal vaginal fistula.
o	Loss of mucus plug.
185
Q

Ix for PROM

A

?USS (amnoiotic fluid level)

Actim-PROM - IGFBP-1 level

High vaginal swab (?infection)

Ferning test - cervical secretion on slide

186
Q

Mx of PROM

A
  • majority fall into labour within 24-48 hours (if >36/40 and don’t then IOL if not in 24h)
  • Monitor for signs of clinical chorioamnionitis and advise no sex

Clindamycin/penicillin during labour if GBS isolated

<36/40 give erythromycin for 10d + corticosteroids

187
Q

Complications of PROM

A
  • Chorioamnionitis
  • Oligohydramnios – if < 24/40 ↑ risk lung hypoplasia.
  • Neonatal death – prematurity, sepsis and pulmonary hypoplasia.
  • Placental abruption
  • Umbilical cord prolapse
188
Q

Examples of rhesus sensitising events

A

<12/40 - Ectopic pregnancy, molar pregnancy, heavy uterine bleeding

<20 /40 - Amniocentesis, CVS, miscarriage, termination of pregnancy; intrauterine death; abdo trauma

> 20/40 - Delivery, Intra-uterine procedures, external cephalic version, APH

189
Q

What is the Bishop score

A

Measure cervical ripeness -

  • ≥ 7 –cervix is ripe – IOL possible
  • <4 –labour unlikely to progress naturally and prostaglandin tablet/gel/pessary required

Features - dilation; length; station (relative to ischial spines); consistency; position

190
Q

Methods of pain relief during labour

A

Non-pharm

  • relaxation/ breathing exercises
  • moving
  • massage
  • music
  • warm bath/ shower

Pharm

  • Entonox (50% O2; 50% NO)
  • Pethidine IM + metoclopramide (opiod)
  • epidural (indwelling plastic catheter) - ideally in 1st stage of labour
  • spinal (c-section/ manual removal of placenta/ instrumental)
  • LA (episiotomy)
  • Pudendal block
191
Q

Mx of abnormal fetal growth pattern

A

serial growth measurements every 2-4 weeks +/- umbilical A doppler (any placental resistance)

modify maternal RF

Corticosteroids

192
Q

RF for abnormal fetal growth

A

LGA
Maternal
- DM; obesity; ↑ age; multiparity; large stature
Fetal
- constitutional; male; post maturity; genetic (Beckwick wierderman -macrosomia + taller than peers during childhood -ch 11)

SGA

  • -hx IUGR; recurrent fetal loss; previous unexplained stillbirth; T1 bleedings; infection; placental pathology (e.g. praevia); multiple pregnancies
  • Smoking; unexplained raised AFP; extremes of maternal age
  • BMI <20; poor nutrition; severe anaemia; High altitude
  • Smoking, alcohol, substance abuse, prescription/ OTC drugs; Domestic violence
  • HTN; haemoglobinopathies; anti-phospholipid syndrome; collagen vascular disease; renal disease
193
Q

Risks of multiple pregnancies

A
Maternal
•Anaemia – ↑ fetal use of maternal blood
• Pre-eclampsia 
• APH/PPH - ↑ trauma
• ↑ sx hyperemesis
•Maternal mortality
Fetal 
• Stillbirth 
• Preterm birth, and associate difficulties (respiratory distress, etc)
• Umbilical cord entanglement
• Feto-fetal transfusion syndrome
194
Q

What is feto-fetal transfusion syndrome and the mx

A

artery-vein anastomosis between twins (usually monozygotic) –> donor becomes anaemic and may show considerable growth retardation; recipient becomes polycythaemic, hypertensive, and will also be much larger

Risk of hydrops foetalis - severe anaemia causing oedema of whole body and HSM

Mx - laser ablation surgery to cut anastomoses or serial amniocentesis (remove membrane so share amniotic fluid)

195
Q

Causes and mx of prolonged 2nd stage of labour

A

Causes
Power –> Inefficient uterine activity;Mx –> Oxytocin (ONLY if 1st baby – if previous uterus can rupture)
- Hypotonic or Incoordinate contractions
- ? dehydration, exhaustion, ketosis –> Mx - hydration, analgesia; amniotomy, oxytocin infusion; delivery if appropriate

Passenger
- Cephalopelvic disproportion (relative, absolute)
- Too big (e.g. macrosomia, hydrocephalus etc.)
- Abnormal diameter presenting (abnormal attitude, malpresentation or malposition)
Mx - C-section; ensure adequate contraction; ?instrumental delivery

Passage
- Abnormal bony pelvis; Cervical dystocia (Mx - C-section) OR Rigid Perineum (Mx – episiotomy)

196
Q

Risks of prolonged labour

A

Chorioamnionitis; 3-4th degree perineal tears; Uterine atony – ↑ the risk of PPH

197
Q

Compare the types of instrumental delivery

A

Ventouse - Attaches cup to fetal skull flexion point (midline, 3cm AP fontanelle) via a vacuum - ↓ success rate. During uterine contractions, traction is applied perpendicular to the cup.

Forceps - Double bladed inserted through pelvis and applied to sides of fetal head
- gentle traction during uterine contractions
+ episiotomy

198
Q

Indications for instrumental delivery

A

Maternal
• Inadequate progress
• Maternal exhaustion.
• Maternal medical conditions - mean active pushing or prolonged exertion should be limited e.g. intracranial pathologies, some maternal congenital heart diseases and severe HTN

Fetal
• Suspected fetal compromise in the 2nd stage of labour, usually diagnosed by:
o CTG monitoring OR Abnormal fetal blood sample
• Clinical concerns e.g significant APH

199
Q

Pre-requisites for instrumental delivery

A

F - Fully dilated cervix
O - Occipito-anterior position (preferably)
R - Ruptured membranes
C - Cephalic presentation
E - Engaged presenting part (head largest diameter should have passed pelvic brim)
P - Pain relief, should be adequate (nerve block / epidural)
S - Sphincter – Bladder empty
o Mediolateral episiotomy - before instrumental delivery - ↓ risk of 3-4th ° tears

200
Q

Ci for instrumental delviery

A
Unengaged
incompletely dilated
cephalo-pelvic disportion
breech/face presentation
preterm/ coag disorder

relative - acute fetal distress/ prolapsed umbilical cord

201
Q

Complications of instrumental delivery

A
> Fetal
Neonatal jaundice
Scalp lacerations
Cephal or sublgaleal haematoma
Facial bruising
Facial N damage
Skull #
Retinal haemorrhage
> Maternal
Vaginal tears
3rd/4th °tears (↑ in forceps)
VTE
Incontinence
PPH
Shoulder dystocia
Infection
202
Q

RF for malpresentation

A

Prematurity; Multiple pregnancy; Uterine abnormalities (e.g fibroids, partial septate uterus, bicornuate uterus); Fetal abnormalities; Placenta praevia; Primiparity; smoking; DM

203
Q

What is used to correct breech presentation, the complications and CI

A

External cephalic version (ECV) ~ 36/40

  • manoeuvring baby through a forward somersault to cephalic presentation from breech
  • some return back (more likely if oligohydramnios, etc)

Complications - fetal distress, PROM, APH, placental abruption, uterine rupture, emergency C-section

CI - recent APH, ruptured membranes, uterine scar/ abnormalities, previous C-section, fetal compromise, vaginal bleeding and medical conditions.

204
Q

How may you pick up on a breech presentation

A
  • Ballotable head in the fundal-area
  • Soft-irregular mass (buttocks) in the pelvis
  • Fetal heartbeat loudest above the umbilicus
  • Sub-costal tenderness
205
Q

Risks of breech presentation

A
  • Premature labour / Cord-prolapse
  • Perinatal mortality 4x ↑
At delivery 
• fetal head entrapment
• cord prolapse
• intracranial haemorrhage (rapid compression head during delivery)
• internal injuries – PROM
206
Q

Presentation and types of a prolapsed cord

A

EMERGENCY - cord passes through as infront of presenting part –> ↓ blood flow - foetal hypoxia and distress

Types

  • incomplete (alongside presenting part but not beyond)
  • complete - desc past presenting part)
207
Q

RF for prolapsed cord

A
  • Transverse/ unstable lie
  • Breech
  • Artificial rupture of membranes
  • Multiple pregnancy
  • Polyhydramnios
  • Prematurity
208
Q

Mx of prolapsed cord

A

HELP
• Avoid handling the cord to reduce vasospasm
• 4-6L oxygen to mother and stop oxytoxin infusion
• Manually elevate the presenting part Lift presenting part off cord by DVE – if in community, fill the maternal bladder with 500ml of normal saline (warmed if possible) via a urinary catheter and arrange immediate hospital transfer.
• Encourage into L lateral position with head down and pillow placed under left hip OR knee-chest position. This will relieve pressure off the cord from the presenting part.
• ? terbutaline - stop contractions, relieving pressure off the cord. Allow time to transfer for delivery
• Emergency C-section

209
Q

Indications for elective C-section

A
  • Breech (at term)
  • Other malpresentations – e.g. unstable (fluctuates ), transverse/ blique lie.
  • Twin pregnancy – when the 1st twin not a cephalic
  • Maternal medical conditions (e.g. cardiomyopathy) –labour dangerous for mum
  • Fetal compromise (e.g. early onset IUGR ± abnormal fetal Dopplers) – thought fetus wouldn’t cope with labour.
  • Transmissible disease (e.g. poorly controlled HIV).
  • 1° genital herpes (HSV) in T3 –no maternal Ab to HSV crossed placenta and protected baby.
  • Placenta praevia
  • Maternal diabetes - fetal weight >4.5 kg.
  • Previous major shoulder dystocia.
  • Previous 3rd/4th perineal tear
  • Maternal request – previous traumatic birth
210
Q

Risks of C-section and how to prevent

A
  • mendelson syndrome (inhale gastric acid contents –> cyanosis, bronchospasm, tachycardia and pulmonary oedema) Mx - H2RA; empty stomach before
  • VTE (stockings +/- LMWH)
  • Bladder injury - catheter
  • supine hypotension - L lateral tilt
  • Pain - epidural/spinal
  • Haemorrhage (PPH/ intra-abdo - G&S

Other: bowel/bladder trauma; ARDA; fetal laceration; infections; fistula; subfertilit

211
Q

Mx of mendelson syndrome

A

HELP; 100% oxygen; Tilt head down; Turn onto left side

  • Aspirate pharynx; Aminophylline and hydrocortisone; ?Abx
  • ± ventilation; PT and rarely bronchoscopic aspiration of mucous plug
212
Q

VBAC v Elective repeat C-section

A

VBAC
- If successful, shorter hospital stay and recovery
- Risk of uterine rupture/ anal sphincter injury
- Risk of maternal death – 4 in 100,000
If successful – good chance of successful future VBACs
- 2-3% risk of transient respiratory difficulties for neonate
- Risk of hypoxic ischaemic encephalopathy (HIE) to the neonate – 0.08%
- Risk of stillbirth beyond 39 weeks whilst awaiting spontaneous labour – 0.1%

C-section

  • Longer recovery
  • ↑ rates of: infection, pain after surgery, fertility issues and need for hysterectomy
  • Risk of maternal death – 13 in 100,000
  • Subsequent pregnancies likely to require C- section
  • 4-5% risk of neonatal respiratory morbidity
  • <0.01% risk of neonatal HIE
  • risk of placental problems (including accreta and praevia), and adhesion formation
213
Q

CI to VBAC

A

absolute - classical caesarean scar, previous uterine rupture, CI for vaginal birth, IOL

Relative - complex uterine scars or >2 prior lower segment Caesarean sections- ↑ risk of uterine rupture

214
Q

Main cause and RF for fetal distress

A

Utero-placental insufficiency –> inadequate oxygen / nutrient supply

RF
• Pre-eclampsia
• Oligohydramnios
• Infection 
• Uterine hyperstimulation	
• Rhesus sensitisation
• Smoking
• IUGR
• ↓ BP (?lying on back/ epidural)
215
Q

Presentation and Ix for fetal distress

A
  • Reduced fetal movement / growth slowing
  • USS - Lack of growth suggests distress / poor prognosis
  • Oligohydramnios / polyhydramnios are associated with fetal distress
  • Meconium stained liquor (risk chemical pneumonitis)

Ix - Dopper USS (↑ placentral resistance
- CTG (late decel?)

216
Q

Mx of fetal distress

A
  • L lateral tilt
  • stop oxytoxics
  • CTG monitoring + FBS (esp if meconium)
    ? C-section <30mins if threat to life
217
Q

Indications for IOL

A
  • 40 -42/ 40 avoid the risks of fetal compromise and stillbirth
  • Maternal - HTN, pre-eclampsia, diabetes and obstetric cholestasis.
  • FGR
  • intrauterine fetal death -IOL offered if mother is physically well with intact membranes.
  • PROM >37/40 if >24 hr
218
Q

CI for IOL

A
>Absolute
Cephalopelvic disproportion
Major placenta praevia
Vasa praevia
Cord prolapse
Transverse lie
Active primary genital herpes
Previous classical C- section

> Relative
Breech presentation
Triplet or higher order pregnancy
2+ previous low transverse C- sections

219
Q

Methods of IOL

A

1) Vaginal PG - ripening cervix+ contract SM - monitor on CTG
2) membrane sweep (>40weeks) - aim to separate chorionic membrane from decidua - release natural PG to kick-start labour
3) Amniotomy - rupture membranes artificially using amnihook - release PG (only if cervix ‘ripe’)

220
Q

Complications of IOL

A
  • Failure of induction (15%)
  • Uterine hyperstimulation – contractions last too long/ frequently, leading to fetal distress - Mx =terbutaline.
  • Cord prolapse
  • Infection – risk is ↓ by using pessary vs tablet/gel,
  • Pain – IOL is often more painful than spontaneous labour. Often epidural analgesia is required.
  • ↑ rate of further intervention vs spontaneous labour
  • Uterine rupture (rare)
221
Q

What is an APH and the causes

A
  • bleeding from >24/40
    Minor <50ml; Major -50-1000ml; massive >1000ml
Causes
•	Placental abruption
•	Vasa praevia
•	Uterine rupture
•	Placenta praevia
•	Marginal placental bleed ( not large enough to cause maternal or fetal compromise)

Local causes
• Benign or malignant lesions – e.g. polyps, carcinoma. cervical ectropion(common)
• Infections – e.g. candida, bacterial vaginosis/ chlamydia
• Vaginal bleeding – trauma/ assault
• Vulval bleeding – lesion/ warts/ trauma

222
Q

Ix and Mx for APH

A

Ix
Bloods -
- FBC (anaemia); clotting profile; G&S; Cross match (min 4 units); U&E + LFT (exclude pre-eclampsia and HELLP)
- Kleihauer test (if Rhesus -ve  determine amount feto-maternal haemorrhage + anti-D required)
- >26/40 - CTG
- USS - Placenta praevia / abruption ?(retroplacental haematoma)

Mx

  • ABCDE and emergency management – Mothers life takes priority
  • IVI + Transfusion – to resuscitate mother
  • Prophylactic Anti-D Ig + Corticosteroids – to prevent rhesus incompatibility or fetal RDS
  • Emergency C-section – if the fetus is alive
223
Q

Pathophysiology and features of a cervical ectropion

A

Benign eversion of the endocervix, exposing the columnar epithelium to the vaginal milieu.
The stratified squamous cells of the ectocervix undergo metaplastic change- simple columnar epithelium (same as endocervix) that contain mucus-secreting glands

Features
Often asymptomatic
 ↑ vaginal discharge (as↑ mucus-secreting glands)
 Post-coital bleeding or intermenstrual bleeding (fine blood vessels present within the epithelium are easily broken during intercourse)
Change is thought to be induced by high levels of oestrogen (so increased risk if COCP, pregnant, menstruating age)

224
Q

Ix and Mx of cervical ectropion

A

Ix –> clinical diagnosis
• Pregnancy test
• Speculum - everted epithelium redish; usually arranged in ring around external os.
• Triple swabs – if any suggestion of infection - endocervical and high vaginal swabs
• Cervical smear – rule out CIN
Mx - No treatment unless symptomatic
1) stop any oestrogen containing medications (e.g. COCP)
2) cryotherapy/ electrocautery = ablate columnar epithelium;
SE - ↑ vaginal discharge until healing complete
3) Medication to acidify the vaginal pH has been suggested, such as boric acid pessaries.

225
Q

Causes and different presentationg of jaundice in pregnancy

A

o Viral hepatitis (40%)

o Obstetric cholestasis (20%) - onset >20/40
 Intense pruritus ± excoriation, no skin rash
•Worse at night, can affect sleep
•↑ AST + ALT + GGT

o Acute fatty degeneration of the Liver
 ↑ bilirubin, ALT + AST
 vom, abdo pain, headache
 Mx - immediate delivery, and treatment of liver failure

Other: gallstones; EBV; CMV

226
Q

Mx of Obstetric cholestasis

A

Monitor LFTs weekly
o Ursodeoxycholic acid (UDCA) – ↓ bile acid levels - ↓ pruritus + improving LFT
o Vitamin K – offered if prolongation of the PT

Delivery >37/40 with CTG in labour.
Check LFTs after 6 weeks.

227
Q

What is Chorioamnionitis and its presentation

A

Bacterial infection of the fetal amnion and chorion membranes - typically ascending GBS infection (or prolonged labour)

Sx
Maternal fever (>37.8), and >2 of the following:
• Leucocytosis (>15,000 /uL)
• Maternal HR >100bpm
• Fetal HR >160 bpm 
• Uterine tenderness
228
Q

Ix and Mx of Chorioamnionitis

A

Ix
•Bloods – FBC, WCC, CRP, cultures – ?sepsis markers
• Aspiration of Amniotic fluid – MC&S
• GBS screen

Mx
o Empiric maternal ABx therapy - Ampicillin + Gentamicin
o C-section – may be indicated to expedite delivery

229
Q

Pathophysiology and types of placental abruption

A

Part/all of placenta separates from wall of uterus prematurely

Blood collects between placenta and uterus –> splits placental attachment from the basal layer.
Detached portion of placenta is unable to function –> rapid fetal compromise.

Types
o Revealed (80%) – bleeding tracks down + drains through the cervix --> vaginal bleeding.
o Concealed (20%)– bleeding remains in uterus (forms clot retroplacentally) --> Bleeding is not visible, can be severe enough to cause systemic shock. (usually incomplete detachment)
230
Q

RF for placental abruption

A
  • Previous abruption
  • Multiple pregnancy
  • Abnormal lie e.g. transverse
  • Polyhydramnios
  • Abdominal trauma
  • Underlying thrombophilias
  • Trauma
  • Pre-eclampsia/ HTN
  • Smoking or drug use
  • Bleeding in t1, particularly if a haematoma is seen inside the uterus on a T1 scan.
231
Q

Presentation of placental abruption

A

Sudden unexpected painful PV bleeding

Tense, tender uterus with a ‘woody’ feel; painful on palpation and difficult to feel fetal parts

CTG – shows bradycardia / asystole (if dead)

232
Q

Pathophysiology and grades of placenta praevia

A

Placenta is fully/ partially attached to the lower uterine segment.
- Identified at 20-week USS (even if asymptomatic):

a. Minor placenta praevia – placenta is low but does not cover the internal cervical os –> repeat scan 36/40 as placenta likely to have moved superiorly
b. Major placenta praevia – placenta lies over the internal cervical os –> repeat scan at 32/40 and make delivery plan (as placenta may move upwards until later on in pregnancy)

Grades
o 4 - Completely covers cervix
o 3 – partially covers
o 2 – reaches internal os
o1 – doesn’t reach cervix but within 2cm of internal os
233
Q

RF for Placenta praevia

A
  • Previous placenta praevia
  • Previous C-section (↑ as ↑ no.)
  • Multiple pregnancy
  • Hx uterine infection (endometritis)
  • High parity
  • Maternal age >40 years
  • Curettage to endometrium after miscarriage or termination
234
Q

Presentation of placental praevia

A
Most commonly:
• Painless PV bleed >28/40
- Usually sudden + heavy 
-Spontaneous / mild trauma
- Does not last for long
• Associated pre-term delivery	
•?  incidental finding on USS
•Abnormal fetal lie (placenta blocks the pelvic inlet
- ~10% have abdo pain
235
Q

Ix and Mx of placenta praevia

A
  • Antenatal USS - confirmed using TV USS/ Abdo USS
  • Uterus should be shoft, non-tender
  • Bloods – Cross match, in preparation of potential APH

NO vaginal exam -may exacerbate APH

Mx
- Minor - ?vaginal deliery
- Major - C- section at 38/40
If APH –> follow that mx and consent for hysterectomy

236
Q

Presentation of placenta accreta

A

Placenta is morbidly attached to the uterine wall to an ↑ degree
o Chorionic villi penetrate the decidua basalis to attach to the myometrium

–> Failure to deliver the placenta in the 3rd stage of labour

237
Q

RF, Ix and Mx of placenta accreta

A

RF
• Previous C-section
• Placenta praevia
• Advanced maternal age

Ix
- Ante-natal USS

Mx

  • methotrexate after delivery (destroy tissue for expulsion
  • elective C-section and hysterectomy
238
Q

Presentation of vasa praevia

A

Vessels run near the internal cervical os

Classic triad
- Membrane rupture (causes rupture of umbilical cord vessels); Painless PV bleeding; Fetal Compromise (as loss of fetal blood)

239
Q

RF, Ix and Mx of vasa praevia

A

RF
• IVF pregnancy
• Multiple pregnancy
• T2 placenta praevia

Ix
- TV Doppler ultrasound –> assess blood flow from placenta

Mx
- delivery at 35-36/40 (before rupture of membranes)

240
Q

Pathophysiology and RF for uterine rupture

A

Full-thickness disruption of the uterine muscle and overlying serosa. The fetus can be extruded from the uterus, resulting in fetal hypoxia and large internal maternal haemorrhage.
• Incomplete – peritoneum overlying uterus is intact. Uterine contents remain within uterus.
• Complete – peritoneum also torn, uterine contents can escape into peritoneal cavity.

RF
• Previous C- section (↑ if vertical incision)
• Previous uterine surgery – such as myomectomy.
• Induction – (Esp wPG ) or augmentation of labour.
• Obstruction of labour
• Multiparity

241
Q

Features of uterine rupture

A

o sudden severe abdominal pain, persists between contractions
o shoulder-tip pain
o± vaginal bleeding

Examination
o regression of presenting part
o scar tenderness and palpable fetal parts.
o Significant haemorrhage –> hypovolaemic shock (↑ HR and hypotension)

242
Q

Ix and Mx of uterine rupture

A

Ix
- CTG (? late decel/prolonged fetal bradycardia are early indicators).
o pathological CTG prompts an emergency section
- ?Maternal haematuria
-?USS if pre-labour
o abnormal fetal lie or presentation, haemoperitoneum and absent uterine wall.

Mx

  • A-E + give blood +/- massive haem protocol
  • C-section <30mins +/- hysterectomy/repair
243
Q

Mx of VTE in pregnancy

A
  • D-dimer not useful as already raised in pregnancy
  • TEDs
  • LMWH and Rivaroxaban (6-12weeks post partum)
244
Q

VTE prophylaxis in pregnancy

A

Measure score using risk assesssment tool

  • > 4 prophylaxis from T1
  • > 3 prophylaxis from 28/40
245
Q

What is lochia

A

Vaginal discharge after giving birth - Starts around 3-4 days after delivery, and continues for 4-6 weeks
starts red-brown - (blood and trophoblastic tissue) then changes to yellow

if offensive odour/ green - infection

246
Q

What is the different types of a retained plancenta and the mx

A

Takes >30mins to deliver:
• Placenta adherens - myometrium fails to contract behind the placenta
• Trapped placenta - detached placenta is trapped behind a closed cervix
• Partial accreta - small area of adherent placenta preventing detachment

Mx - oxytocin IM - produce contractions and expel remaining placenta; if fails –> manual removal under GA

247
Q

What is Sheehan’s syndrome and how does it present

A

Hypo-pituitarism due to anterior pituitary infarction, following PPH

Dx –> ↓ Prolactin + GRH following a hx of PPH
-Lack of lactation; Amenorrhoea; Hypothyroid symptom

248
Q

Mx of sheehans syndrome

A

Life-long hormone replacement therapy managed by endocrinology– to replace ↓ levels, may include:
o Levothyroxine
o LH + FSH
o Corticosteroids

249
Q

What is shoulder dystocia and how does it present

A

After delivery of the head, the anterior shoulder of the fetus becomes impacted on the maternal pubic symphysis, or (less commonly) the posterior shoulder becomes impacted on the sacral promontory. EMERGENCY

Presentation -
• Difficulty delivering the face/chin
• Head retraction (turtle-neck sign) neck no longer visible
• Failure of the shoulders to descend – doesn’t turn to look to side

250
Q

RF for shoulder dystocia

A
Pre-labour
•Previous hx
•Macrosomia 
•DM  (macrosomic - baby’s disproportionately bigger abdomen compared to head)
•Maternal BMI > 30
•IOL
Intrapartum 
•Prolonged 1st or 2nd stage of labour
•2° arrest (initially good progress in labour and then progress stops, usually due to malposition of baby)
• Augmentation of labour with oxytocin
•Assisted vaginal delivery
251
Q

Mx of shoulder dystocia

A

Keep head in line with spine , and do not apply fundal P (↑ risk uterine rupture).
SHEMSPIER
• Stop pushing
• HELP (senior obstetrician &midwife; paediatrician)
• ? Episotomy (make access easier) –
• McRoberts (knees to chest – hyperflex maternal hips) - flatten sacral promontory + ↑ lumbosacral angle + AP diameter
• Suprapubic P – sustained/rocking – behind ant shoulder to disimpact it from underneath maternal symphysis
• Posterior arm delivery
• Internal rotation- (“corkscrew manoeuvre”) - apply P simultaneously in front of one shoulder and behind the other to move baby 180 ° or into an oblique position.
o All fours / repeat
• Emergency
- Cleidotomy – # fetal clavicle; Symphysiotomy – cutting the pubic symphysis.
- Zavenelli – returning fetal head to pelvis for delivery of the baby via C- section.
• Review
- PR examination - exclude a 3rd ° tear
-Debrief mother and birth partner(s), advise them of the risk of recurrence with any subsequent delivery. (very traumatic if no regional anaesthesia)
-PT review – ↑ risk of pelvic floor weakness/3rd degree tear, MSK pain and temporary nerve damage.
-Paediatric review - ? brachial plexus injury, humeral # or hypoxic brain injury.

252
Q

Define PPH

A

Primary - Loss of blood >500ml, within 24 hours of delivery
o Minor <1000 mls
o Major >1000 mls

Secondary - Loss of blood >500ml – 24h- 6weeks (most common 7-10 d) after delivery

253
Q

Causes of Primary PPH

A

1) Tone Uterine atony –> uterus fails to contract adequately following delivery, due to a lack of tone in the uterine muscle.
2) Tissue – Retained placenta / clots – 2nd most common cause (stops uterus contracting)
3) Trauma – uterine, cervical, or vaginal laceration during delivery (↑ risk – instrumental; episiotomy; c-section)

4) Thrombin – Coagulopathy (pre-existing e.g. vWD/ haemophilia or acquired - DIC/HELLP) and vascular abnormalities (Placental abruption, hypertension, pre-eclampsia)
Also risk if anaemic at onset

254
Q

RF for uterine atony

A

Maternal profile: Age >40, BMI > 35, Asian ethnicity.

Uterine over-distension – multiple pregnancy, polyhydramnios, fetal macrosomia.

Labour – induction, prolonged (>12 hours)/ use oxytocin/ GA

Placental problems – placenta praevia, placental abruption, previous PPH

255
Q

Presentation of Primary PPH

A

Clinical features
• Continuous PV bleeding
• Fails to stop after delivery of the placenta
• Dizziness, palpitations, SOB

O/E

  • Shock (↑ HR; ↑ RR; ↓ BP, cyanosis,)
  • Speculum - reveal sites of local trauma causing bleeding.
  • Examine placenta - ensure complete (missing cotyledon/ ragged membranes could cause PPH).
256
Q

Initial Mx of PPH

A

HELP
ABCDE +/- activate massive haemorrhage protocol 2222

  • ? tranfuse 2L warmed crystalloid or o-ve blood
  • X-match 4-6 units

trauma - repair laceration; rupture - repair/ hysterectomy

Tissue - IV oxytoxin and manually remove placenta than prophylactic Abx

Thrombin - blood products

257
Q

Definitive Mx of uterine atony causing PPh

A

Bimanual compression to stimulate uterine contraction – fist insider the ant fornix + P on abdomen at the post aspect of the uterus (ensure the bladder emptied by catheter)

Pharmacological measures – ↑ uterine myometrial contractions e.g. Syntometrine ( Syntocinon + ergometrin)/ carboprost; misoprostol

Surgical measures – intrauterine balloon tamponade, haemostatic suture around uterus (e.g. B-lynch), bilateral uterine or internal iliac artery ligation, hysterectomy (as a last resort).

258
Q

MOA, SE and CI of syntocinon

A

MOA - Synthetic oxytocin, act on oxytocin R in the myometrium to contract uterus (short t ½ so can give as infusion). Given IV/IM.

SE - N&V, headache, rapid infusion, ↓ BP, fluid retention (if large volume)

CI - Hypertonic uterus; uterine spasm\rupture, severe CVS disease (↓ BP/ reflex ↑ HR)

259
Q

MOA, SE and CI of Ergometrine

A

MOA - Multiple receptor sites action – contracts uterus and small blood vessels. Given IV/IM.

SE - HTN, nausea, bradycardia

CI - HTN, eclampsia, vascular disease, glaucoma

260
Q

MOA, SE and CI of Carboprost

A

MOA -. Prostaglandin analogue
(up to 8 doses x 20ml IM)

SE - Bronchospasm, pulmonary oedema, HTN, pulmonary HTN (multiple doses), CV collapse

CI - Cardiac disease, pulmonary disease i.e. asthma, untreated PID

261
Q

MOA and SE of MIsoprostol

A

(PR/Oral) Prostaglandin analogue (v. cheap and don’t keep in fridge)
Diarrhoea, malignant hyperthermia (if >1g given)

262
Q

Causes and features of secondary PPH

A

Causes:
o Retained placenta
o Infection (endometritis) RF - C- section, PROM and long labour
o Abnormal involution of the placental site (inadequate closure and sloughing of the spiral A at the placental attachment site).
o Trophoblastic disease (v. rare)

Clinical features
o Excessive vaginal bleeding
 spotting on-and-off for days after her delivery, occasional gush of fresh blood. OR
 Haemodynamically unstable/ massive bleed  HELP AND RESUSCITATION
 ? Open os – obstetric exploration
o Additional features depend on underlying cause
 Endometriosis –> fever/rigors, lower abdominal pain or foul smelling lochia (the normal discharge from the uterus following childbirth).
o Abdominal examination –> lower abdominal tenderness (? endometritis), or the uterus may still be high (? retained placenta).
o Speculum examination (amount bleeding) + high vaginal swab

263
Q

Ix and Mx of Secondary PPH

A

Ix

  • Bloods –> FBC; U&E; CRP; Coagulation profile; G&S; Blood cultures (if pyrexial)
  • Imaging –> Pelvic USS (?retained placental tissue- good -ve predictive value)
  • High vaginal swab?

Mx
- IV Tazocin ± metronidazole
o + Gentamicin if endomyometritis (tender uterus) or overt sepsis.
- Uterotonics – same 1° PPH ( + ?syntometrine (oxytocin+ ergometrine))

Surgical - if excessive/ continuing bleeding (irrespective of USS findings) -? balloon catheter
o Evacuation of retained POC
 ↑ risk of uterine perforation (uterus softer and thinner post-partum)

264
Q

Presentation and ix findings of HELLP syndrome

A

Haemolysis, Elevated Liver enzymes and Low Platelets
1) AST↑ 2) LDH ↑ 3) blood film – burr cells and polychromasia (haemolysis - MIHA)
• Platelets <100 x 109/L due to activation and ↑ consumption

  • 70% of cases present before delivery, peaking at 27-37 weeks of gestation
  • Usually non-specific symptoms – headache, visual symptoms, fatigue, RUQ / epigastric, pain, N&v
265
Q

Mx of HELLP syndrome

A

Delivery if >34/40

Transfusion of red cells, platelets, FFP and cryoprecipitate or fibrinogen concentrate are required as indicated clinically and by blood tests

266
Q

RF for AFE

A

Amniotic fluid embolism

  • Multiple pregnancy
  • Increasing maternal age
  • Induction of labour
  • Uterine rupture
  • Placenta praevia
  • Placental abruption
  • Cervical laceration
  • Eclampsia
  • Polyhydramnios
  • C- section / instrumental delivery
  • Disruption of vessels supplying uterus
  • Strong uterine contractions (prevent with IV β – adrenergic drugs /magnesium sulphate)
267
Q

Clinical features of amniotic fluid embolism

A
Sudden onset of:
•	Hypoxia/respiratory arrest --> ARDS
•	Hypotension
•	Fetal distress
•	Seizures
•	Shock
•	Confusion
•	Cardiac arrest
•	DIC (may be 1st sign – all within 4 hours)
•	Acute renal failure 
•	Uterine atony (uterus fails to contract --> PPH)
268
Q

Differentials of amniotic fluid embolism

A

PE; anaphylaxis; sepsis; eclampsia; MI

269
Q

Ix and Mx for amniotic fluid embolism

A

ABCDE
 high flow oxygen -
 IV access & fluids
 ± FFP/ platelets

Early MDT involvement
o Anaesthetics –> ITU
 Measure pulmonary artery wedge pressures
oHaematology - Manage DIC

Continuous fetal monitoring
o if pt stable –>delivery
o if cardiac arrest/ maternal compromise–> perimortem section and CPR of mother

If uterine atony - oxytocics

Ix
• Bloods – FBC, U&E, calcium and magnesium, clotting studies, ABG
• ECG (ischaemic changes)
• CXR (shows pulm oedema)

Definitive diagnosis = Post mortem - fetal squamous cells along with debris in the pulmonary vasculature.

270
Q

When to activate the Major obstetric haemorrhage protocol

A

Call 2222:
If >4 units red cells <1hour

> 1500ml blood loss in pregnancy/ labour or following birth of baby

Losing blood >150ml/min

271
Q

Mx of epilepsy in pregnancy

A

Lamotrigine safest + vitmain K for last 4 weeks of pregnancy
+ 5mg of folic acid per day from before conception until 12 weeks of pregnancy to reduce the risk of NTDs

272
Q

How to prevent HIV transmission from mother to baby

A

1) NNRT throughout pregnancy and delivery
2) C-section delivery if detectable viral load
3) No breastfeeding
4) NRTI to baby for 4-6 weeks (Zidovudine)

273
Q

How to mx diabetes in pregnancy

A

Ideally keep HBA1C <8%
+/- metformin

Insulin requirements increase - need regular sugar checks - may need to + / ↑ insulin. Increases risk of hypos and can lose hypo-awareness

Ketones can cross placenta - don’t deliver manage mother and baby to reduce risk

Put on growth scan pathway (macrosomia)

in labour - variable insulin rate infusion - check gluc regulaley

Encourage breast feeding

274
Q

Risks to pregnancy of uncontrolled diabetes

A

miscarriage, congenital malformation, stillbirth, neonatal death, IUGR, pre-eclampsia, prematurity

275
Q

Types and RF for hypertension in pregnancy

A

o Gestational hypertension - pregnancy-induced HTN (PIH) which develops >20 weeks
o Pre-existing hypertension - defined as chronic HTN, either pre-pregnancy/ at booking, <20/40– without proteinuria

RF
•	Previous / pre-existing hypertension
•	CKD	
•	AI disease (SLE, etc)
•	Diabetes
•	Hx pre-eclampsia
•	BMI >35
276
Q

Dx and Ix for HTN in pregnancy

A

Dx - dBP >90 mmHg, on ≥2 occasions, ≥4 hours apart, or a single dBP >110 mmHg

Ix - BP and Urine dip – monitor regularly
• Bloods – FBC, U+E, LFTs – monitor regularly in moderate-severe disease, moderate as a one-off and severe as a weekly test
• Fetal USS - fetal growth and amniotic fluid vol and Doppler velocimetry of umbilical arteries

277
Q

Mx of HTN in pregnancy

A

Aspirin 75mg OD – indicated after 12 weeks in women with RF to prevent pre-eclampsia

Rest and exercise

Treatment –
o Labetalol –> if BP >140/90
Monitor BP 2 x week (+ urine dip if >150/100)

If >160/110 - admit –> BP QDS + urine daily +bloods

alternative = Methyldopa / Nifedipine

278
Q

Criteria for pre-eclampsia

A

1) HTN (sBP >140 or dBP >90) – x2 at least 4 hrs apart Or dBP >110
2) Significant proteinuria
o 2+ protein on urinalysis – prompt a urinary protein:cr
o >30 mg/mmol urinary protein:cr – prompt 24 hr
o ≥ 300 mg protein in a 24-hr urine sample
3) In a woman >20/40

279
Q

Define eclampsia

A

Generalized convulsions during pregnancy, labour or ≤ 7 days postpartum & not caused by epilepsy/ other neurological disorder such as epilepsy or metabolic causes
o ≥1 convulsions superimposed on pre-eclampsia
o Last~ 60 to 75 secs, followed by a variable lasting post-ictal phase.
oMaternal convulsions may cause fetal distress; bradycardia and neurological damage induced by hypoxia

280
Q

RF for pre-eclampsia

A
Moderate 
•	Nuliparity.
•	Maternal age ≥ 40 years.
•	Maternal BMI ≥ 35 at initial presentation.
•	FH of pre-eclampsia.
•	Pregnancy interval > 10 years.
•	Multiple pregnancy	

High
• Chronic hypertension
• HTN, pre-eclampsia or eclampsia in previous pregnancy.
• Pre-existing CKD
• Diabetes Mellitus
• AI disease (e.g. SLE, antiphospholipid)

281
Q

Presentation of pre-eclampsia

A
  • Hew onset HTN >20/40
  • New onset proteinuria
  • Headache (frontal)
  • New oedema (face, hands, feet) (due to ↓ oncotic P)
  • Agitation
  • Hyperreflexia/ clonus
  • Epigastric pain (due to hepatic capsule distention ± vomiting
  • IUGR on ante-natal USS
  • HELLP syndrome
  • Visual disturbances e.g. blurred or double vision, halos, flashing lights –> Retinal oedema, haemorrhage or papilloedema
282
Q

Ix of pre-eclampsia

A

Regular BP monitoring
Urinalysis - MC&S
24hr urine collection
Fetal USS +Doppler of umbilical A

Monitor for signs of organ dysfunction/ HELLP/ reversible causes:
FBC (↓ Hb, ↓ platelets); U&E: (↑ urea, creatinine, urate, ↓ urine output);LFT (↑ ALT & AST); Clotting studies (?DIC); Blood glucose (reversible cause of eclampsia)

283
Q

Additionally ix for eclampsia

A

CT/MRI head
o HTN/ pre-eclampsia who has focal neurology; not recovered from seizure
o Hx head trauma; Seizure onset in T1

Abdominal USS – estimate gestation age and rule out placental abruption;

CTG monitoring – evidence fetal distress/ bradycardia

284
Q

Complications of pre-eclampsia/ eclampsia

A

Maternal
• HELLP syndrome
• fibrin deposit in hepatic sinusoids/ Hepatic rupture
• DIC
• Glomerular damage leading to proteinuria, hyperproteinaemia + ↓ oncotic pressure (which exacerbates hypovolaemia). ?AKI ± cortical necrosis
• ARDS/ Pulmonary oedema
• Cerebrovascular haemorrhage -CNS thrombosis and fibrinoid necrosis of cerebral A –> Permanent CNS damage/ oedema ± cortical blindness
• Hypertension
• Death

Fetal Complications
• IUGR ( impaired uteroplacental circulation); Intrauterine fetal death.
• Prematurity (iatrogenic and idiopathic)
• Infant respiratory distress syndrome; Placental abruption

285
Q

Mx of pre-eclampsia

A

Admit if any sign of proteinuria

  • limit fluids to 80ml/hour
  • control BP
  • Magnesium sulfate (reduce risk eclampsia)
  • delivery of fetus (c-section if severe)
286
Q

Mx of eclampsia

A

ABCDE assessment and intervention as appropriate.
Pt lie in the left lateral position, with secured airway and oxygen therapy.

Magnesium sulphate; + continued 24h after delivery OR last seizure (whichever later)- stop seizure and prevent
- assess for signs of ↑ Mg (hyper-reflexia, respiratory depression), and the fetus monitored via continuous CTG.

IV labetalol + hydralazine. A target MAP of <120mmHg.
- continuous CTG during and for 30 minutes after

Delivery of Baby and Placenta (definitive treatment) – ideally C-section ASAP
Mother - seizures controlled, severe HTN treated and hypoxia corrected before

HDU/ ITU at least 24hours after until - well controlled BP (<150/100), adequate UO and discontinuation of MgSO4

Monitor fluid balance monitoring - prevent pulmonary oedema and detect AKI.
After 72h measure FBC, LFT and creatinine.
Monitor BP (daily 2 weeks) for 6 weeks after. + check bloods and urine at 6 weeks.

287
Q

Differentials of eclampsia

A
Hypoglycaemia.	Medication-induced.
Pre-existing epilepsy.	
Brain tumour.
Head trauma.	
Cerebral aneurysm.
Haemorrhagic stroke.	Septic shock.
Meningitis.	
Ischaemic stroke.
288
Q

SE of Labetalol

A

Β-blocker.

SE –> Postural hypotension, fatigue, headache, nausea and vomiting, epigastric pain.

CI- asthma; T1DM; pheo

289
Q

Pathophysiology of pre-eclampsia

A

Trophoblasts fail to invade spiral arteries - remodelling of spiral arteries incomlete leading to a high resistance, low-flow uteroplacental circulation –> ↑ BP, hypoxia and oxidative stress

290
Q

Define gestational diabetes and mx

A

Glucose intolerance with onset during pregnancy - usually T3 (if in T1 - DM)

Fasting glucose > 5.6mmol/L
2hrs postprandial glucose > 7.8mmol/L

Mx - Diet control ± metformin ± insulin

291
Q

RF for gestational diabetes

A
  • BMI >30
  • Asian ethnicity
  • Previous hx
  • 1st degree relative DM
  • PCOS
  • Previous macrosomic baby
  • Polyhydramnios
  • Multiple pregnancies
  • Unexplained still birth
292
Q

RF for perinatal mental health problems

A
  • < 16 y/o
  • Unrealistic ideas of motherhood
  • Pre-existing MH e.g. schizophrenia, OCD, bipolar
  • FH (1st ° relative) of MH
  • Volatile or Absent family relationships
  • Social Isolation
  • Lack of positive supportive partner
  • Poor or inadequate antenatal care
  • Pregnancy Complications
Social problems
• Hx substance misuse
• Migrant, asylum seeker or refugee 
• Difficultly speaking English
• Experienced domestic abuse
• Homeless
293
Q

Define postnatal depression

A

Depressive episode within the first 12 months postpartum

294
Q

Presentation of post-partum psychosis

A

Onset often <3m of delivery

  • Appear confused and distracted/restless
  • Quiet and withdrawn OR agitated/irritable and distressed
  • Express bizarre ideas – paranoid/grandiose delusions, report/respond to auditory hallucinations.
  • Appear manic- talking rapidly and being more active.
  • Sleep disturbance
295
Q

Mx of post-partum psychosis

A

Prompt assessment by specialist MH services
o Risk assessment - risk to mother and child
 suicidal ideation, thoughts of harm to or bizarre ideas about the baby, self-neglect and ability to provide care for the baby

Most treated as inpatient – specialist mother and baby units OR general psychiatric ward
 Anti-psychotics e.g. olanzapine ± mood stabiliser (lithium); ?ECT in severe

Close monitoring during future pregnancies and after delivery

296
Q

RF for STIs

A
  • minority ethnic groups
  • low socio-economic status
  • poor education
  • unemployed people
  • multiple of sexual partners
  • age at first sexual intercourse
  • concurrent partners
  • sexual orientation
  • non-barrier contraception
  • Co-infection with another STI
  • +ve sexual partner
  • Men who have sex with men (MSM).
297
Q

Presentation of vulvovaginal candidiasis

A

Symptoms

  • Pruritus (itchy) vulvae–can extend to perineal region
  • Vaginal discharge - white, cottage cheese – like and non-offensive Dysuria (superficial dysuria)

Signs

  • Erythema and swelling of the vulva
  • Satellite lesions
  • Red, pustular lesions with superficial white/creamy pseudomembranous plaques that can be scraped off.
298
Q

RF and organism responsible for vulvovaginal candidiasis

A
  • Pregnancy (↑ glycogen production - more favourable environment for mo )
  • Abx (broad spectrum – alter vaginal micro-biota) • Diabetes
  • Immunodeficiency (HIV/Ca)
  • Steroids

Caused by candida albicans

299
Q

Mx of vulvovaginal candidiasis

A

OTC –>
- Clotrimazole (via applicator) –> esp in pregnancy
- PO fluconazole -50mg OD for 2/52
± Topical imidazole (address vulval symptoms) – ?weaken latex condoms

Children - nystatin

If fail to clear in 1-2 weeks - measure vaginal Ph and swab. Mx RF.

Advice - soap substitutes; avoid cleaning vaginal area >1/day; avoid potential irritants (e.g. shower gels, vaginal deodorants, douches) and tight fitting underwear/tights.

300
Q

Pathophysiology and RF for bacterial vaginosis

A

Normal vaginal flora disturbed –>↓ no lactobacilli (produce H2O2 -keep pH vagina <4.5 - inhibiting the growth of other mo)
subsequent ↑ in vaginal pH – growth other mo (Gardnerella vaginalis, anaerobes and mycoplasmas)

Risk factors - IUD, vaginal douching, smoking, recent abx, presence STI

301
Q

Presentation of bacterial vaginosis

A

50% asymptomatic

Smelly, thin, white-fishy discharge (Most common cause of abnormal vaginal discharge in childbearing age.)

302
Q

Ix for bacterial vaginosis

A

High vaginal smear:

  • clue cells
  • ↓ no. lactobacilli
  • Absence of pus cells

Swab for STI

+/- Vaginal pH >4.5 ± KOH (potassium hydroxide) whiff test – + KOH to discharge - strong fishy odour

303
Q

Mx of Bacterial vaginosis

A

Metronidazole PO – 400 BD for 5-7/7 – or 2g stat dose OR

Clindamycin 2% cream – ON for 7/7

Advice - avoid vaginal douching, scented shower gels, antiseptic agents and shampoos in the bath, Removal of an IUD. Can recur (>50% in <3m)

304
Q

Pathophysiology of chlamydia

A

Caused by Chlamydia trachomatis an obligate intracellular gram -ve bacterium - invades the cells of the endocervix, urethra, fallopian tubes, rectum, and conjunctivae .

Transmitted -. unprotected vaginal, anal or oral sex

Highly infectious, and infects in >50% of exposures – incubation period 7-21d

Commonly associated with Gonorrhoea

305
Q

Presentation of chlamydia in males

A
Males ~50% asymptomatic
Symptoms
o	Urethritis --> Dysuria; Urethral discharge
o	Epididymo-orchitis
o	Testicular pain (prostatitis)

Signs
o Epididymal tenderness
o Mucopurulent discharge

306
Q

Presentation of chlamydia in females

A
Females ~70% asymptomatic
Symptoms 
o	Dysuria
o	Abnormal vaginal discharge
o	Intermenstrual/ postcoital bleeding
o	Deep dyspareunia/ Lower abdominal pain
o	Salpingitis 
Signs
o	Cervicitis +/- contact bleeding
o	Mucopurulent endocervical discharge
o	Pelvic tenderness/ Cervical excitation
307
Q

Ix for chlamydia and gonnorhoea

A

Women: Vulvo-vaginal swab (1st ) or endocervical swab - NAAT
Men: first catch urine sample (first choice) or urethral swab - NAAT

Swabs from rectum, eye, throat too
+/- PCR

308
Q

Complications and risks in pregnancy of chlamydia

A
Males
• Epididymitis/ epididymo-orchitis
• Infertility
• Proctitis
• Reiters disease
Females
• PID (infertility)
• Salpingitis ± endometriosis 
• Perihepatitis
• Ectopic pregnancy
Pregnancy --> premature, low birth weight, miscarriage, stillbirth, transfer to neonate in delivery causing conjunctivitis/ pneumonia)
309
Q

Mx of chlamydia

A

Doxycycline – 100mg BD 7/7 (CI in pregnancy)
OR
Azithromycin – 1g one-off

Erythromycin in neonates

Repeat testing recommended in 3m.

310
Q

Pathophysiology of gonorrhoea

A

Neisseria gonorrhoeae- Gram–ve diplococcus bacterium.
• 2-14d incubation, sx~2-5 days after infection

Transmission- unprotected vaginal/oral/anal sex + Vertical

Strong affinity for mucous membranes - Uterus, urethra, cervix, fallopian tubes, ovaries, testicles, rectum, pharynx and conjunctiva

Pili that attach -> endocytosed into cell -> acute inflammation.

Surface proteins -> bind R of immune cells + prevent immune response

311
Q

Presentation of gonorrhoea

A
Males - 10% asymptomatic
 Urethritis – Mucopurulent urethral discharge
 Dysuria
 Proctitis (MSM)
 Epididymal tenderness

Females - 50% asymp
 Altered/↑ vaginal discharge (thin, watery, green or yellow)
 Dysuria; Dyspareunia; Lower abdominal pain
 Rare – intermenstrual ± post-coital bleeding
 Easily induced cervical bleeding
 Pelvic tenderness

Rectal - ? anal discharge +/- pain/discomfoty

312
Q

Complications (+ in pregnancy) of gonorrhoea

A

Males
• Epididymitis
• Prostatitis

Females
• Bartholins abscess – severe unilateral labial pain
• Fitz-Hugh-Curtis syndrome
• Endocervivitis, urethritis, salpingitis
• PID - chronic pain, infertility and ectopic pregnancy

Both - Disseminated gonococcal infection - malaise, joint pain, fever, rash

Pregnancy - Perinatal mortality, spontaneous abortion, premature labour, early fetal membrane rupture. Vertically transmission during delivery - neonatal gonococcal conjunctivitis

313
Q

Mx of gonorrhoea

A

Ceftriaxone 500 mg IM stat + Azithromycin 1 g orally stat

Culture should be performed at 72 hours, as test of cure

314
Q

Pathophysiology of HSV

A

HSV-1 - Cold sores (lasts 7-10 days) ± genital herpes (oral sex)

HSV-2 - genital herpes.

Spread - via skin-to-skin contact - travels up the nearest nerve to ganglion and remains - dormant (can’t be reached by immune system)
- During reactivation - travels back down the nerve onto the surface of the genitals once again to cause a symptomatic outbreak.

Asymptomatic shedding - can transmit (may be unaware have infection)

315
Q

Presentation of HSV

A

Primary infection may be asymptomatic

1° infection - Painful small red blisters around genitals, can form open sores
o penis/vulva/clitoris, anus, buttocks and thighs
o Vaginal or penile discharge ± itch
o Flu-like symptoms, fever, muscle aches, dysuria, Inguinal lymphadenopathy
After around 20 days the lesions crust and heal.

2°/recurrent - outbreaks often shorter and less severe over time (as Ab ↑ recognition of the virus and ↑ the effectiveness of response) -↑ if HIV
- Painful red blisters; Burning & itching around the genitals
o Viral shredding only occurs with these lesions and ulcerations

If contract in T3 no Ab cross placenta so high risk of child getting.

316
Q

Ix and Mx for HSV

A

Swab ulcer - PCR / serology

Mx - no cure; avoid sex in outbreak; contact trace; painkillers; ice packs

?Aciclovir 400 mg TDS – within 5d of sx - ↓ severity and duration (if recurrent take as soon as get sx)
>6 outbreaks/year – take prophylactically

317
Q

Presentation of neonatal herpes

A

Skin, eyes and mouth (SEM) herpes –> antivirals

Disseminated (DIS) herpes  internal organs (↑ mortality)

CNS  encephalitis (↑ mortality)

318
Q

Pathophysiology of syphilis

A

Treponema pallidum - Spirochete gram-ve bacterium – replicates slowly

Enters through a break in the skin or through intact mucous membranes

Can be transmitted through infected blood products

Commonly occurs with HIV

319
Q

Presentation of syphilis (1° and 2 °)

A

1° lesion - papule will appear before ulcerating into a chancre (infectious hard painless, surrounded by bright-red margin) develop at site of contact
2 ° stage ~6 wks after 1° lesion - pathogen disseminates - highly infectious
o Fever/ Myalgia/ malaise/ arthralgia; Sore throat; Headache
o Skin rash (hands & soles of the feet – not itchy/painful)
o Weight loss
o Condylomata lata- elevated plaques like warts at moist areas of skin e.g. inner thighs, anogenital region, axillae
o Painless lymphadenopathy
o Silvery-gray mucous membrane lesions - oral, pharyngeal, genital

Latent period  patient can remain asymptomatic for months to years

320
Q

Presentation of tertiary syphilis

A
  • obliterating arteritis

Gummatous syphilis – locally destructive inflammatory nodules
- Granulomas - bone, skin, mucous membranes of upper resp tract, mouth and viscera or CT e.g. lung, liver, testis.

Neurosyphilis:
- Tabes dorsalis; Dementia; Meningovascular complications (CN palsies, stroke, cerebral gummas); Argyll Robertson pupil – pupil is constricted and unreactive to light, but reacts to accommodation

Cardiovascular syphilis:
- AR due to aortic valvulitis + aortic root dilatation; Angina (stenosis of the coronary ostia); Dilation and calcification of the ascending aorta.

321
Q

Ix for syphilis

A

Dark field microscopy of chancre fluid- detects spirochaete in 1° syphilis
• PCR testing of swab from active lesion

• Serology:
o Treponemal tests – assess for exposure to treponemes
 Treponemal ELISA (IgG/IgM) – +ve for life
 TPPA or TPHA –+ve for life
o Non-treponemal tests
- RPR/VDRL: ↑ in early disease; ↓ titres indicate successful treatment or progression to late disease. False +ve - inflammatory conditions/ pregnancy.

Lumbar puncture: CSF antibody tests in neurosyphilis

322
Q

Mx of syphilis

A

1st line - PenV IM STAT (early – 1 dose); late (weekly) (neuro – IV)
2nd line - Azithromycin stat

  • Tertirary –> oral steroids prior to abx (↓risk of an acute localised inflammatory reaction)
  • risk of Jarisch Herxheimer reaction - inflammatory response 2° to death of treponemes, and results in a flu-like illness within 24 hrs of treatment - Supportive measures

Follow-up serology to determine response to treatment

323
Q

Risks of syphilis in pregnancy

A

Potential cross placenta/ infect during delivery (congenital) - treat - ↓ risk of miscarriage, stillbirth, pre-term labour or congenital syphilis.

Congenital syphilis
- severe + debilitating - saddle nose, rashes, fever, failure to gain weight.

324
Q

Pathophysiology of Trichomonas Vaginalis

A

Anaerobic flagellated protozoan spread by vaginal sexual intercourse
Replicates via binary fission

325
Q

Presentation and risks in pregnancy of Trichomonas Vaginalis

A

Male - Usually asymptomatic
• Urethritis - Urethral discharge & Dysuria
• Urinary frequency
• Pain/ itch around foreskin
• Balanoposthitis – inflammation of the glans penis (rare)
• prostatitis
Female
• ~70% have vaginal discharge (smelly, frothy, yellow-green-ish)
• Itchy/ sore vulva; Vulvitis; Vaginitis
• Dyspareunia; Dysuria;
• Strawberry cervix (cervicitis)– punctate and papilliform appearance (due to small petechial haemorrhages)
pregnancy –> Premature labour; low birth-weight; predispose to maternal postpartum sepsis.

326
Q

Ix for Trichomonas Vaginalis

A

Female: - High vaginal or Self-administered vaginal swab

Males: Urethral swab or first void urine sample

Swabs –> NAAT +/- MCS

Bloods - if systemic features of infection.

327
Q

Mx of Trichomonas Vaginalis

A

Metronidazole 2g PO stat, OR 400mg BD for 5-7/7 (no alcohol) Avoid breastfeeding 24h.

328
Q

Presentation of HPV (6&11)

A

Many asymptomatic

Warts - penis, scrotum, vulva, inside vagina or anus, cervix, perianal skin (speculum)

Usually painless, fleshy growths that can be soft/hard & singular/ multiple

Occasionally warts may cause irritation or become inflamed.

Extra-genital lesions - oral cavity, larynx, conjunctivae and nasal cavity

329
Q

Mx for Genital HPV warts

A

Watch and wait – 1/3 regress spontaneously within 6 months

Soft non-keratinised lesions – Podophyllotoxin 0.15% cream BD for 3/7 then 4d rest, repeated for 5 wks (CI - pregnancy, breastfeeding, weakens condoms)

Hard keratinised lesions – Imiquimod 5% cream – apply 3/7 and wash off after 6-10 hours - max of 16 weeks

Resistant/recurrent/ in pregnancy– Ablation, Cryotherapy or excision should be used

Screen for other STIs

330
Q

Pathophysiology of HIV

A

HIV –> CD4 R on Th –> penetrates –> combines host dna –> makes viral protein and virus assembles –> disseminates to secondary lymphoid tissue –> damaged

331
Q

Stages and sx of HIV infection

A

Sero-conversion (start to produce anti-HIV ab) - 1-6wk
- lymphadenopathy/ glandular fever like, myalgia, pharyngitis, headache, diarrhoea, neuralgia, neuropathy, fever, malaise, macpap rash

Asymptomatic infection
- m-years

Persistent generalised lymph-adenopathy (PGL)
- nodes >1 cm at 2 extra-inguinal sites, last >3m

Symptomatic infection
- fever, night sweats, diarrhoea, weight loss
+/- opportunistic - HZV, candida, HSV, tinea
- lasts years
AIDS
- severe immunodef –> CD4 count <200 x 109 /L
- life threatening infection/tumours-
p. jiroveci, non-Hodgkin’s lymphoma, and TB

332
Q

Ix and monitoring for HIV

A

ELISA (serum/saliva) - HIV ab and p24Ag –> +ve 4-6 weeks after exposure
- produce IgG+IgM after 3m

Contact tracing

STI screen

Baseline CXR

Pregnancy test

Monitor –> CD4 count; HIV viral load (Viral Ag & HIV RNA) - aim HIV RNA <20 /mL
FBC (?pancytopenia); U&Es; Urinalysis; ALT, AST and bilirubin

333
Q

Mx of HIV

A
  • counselling
  • sexual health advice
  • High activity anti-retroviral therapy (HAART) >3 drugs starting ASAP
  • NRTI/ NNRTI/ inSTI/ PI
    e.g. Atripla: tenofovir (NRTI) + emtricitabine (NRTI) + efavirenz (NNRTI)
    SE NRTI and PI- myalgia, neuropathy, pancreatitis
    NNRTi - hepatitis, dizzy
334
Q

When to use PEP

A

Has/ suspects been exposed to the virus within the last 72 hours to ↓ risk of becoming infected. A PEP course lasts for a month:
Truvada (one tablet daily) + Raltegravir (one tablet twice daily)

Check patient risk – related to donor CD4 + HIV RNA levels

Needle-stick injury transmission <0.5% (bloods at 3, 6 + 8m to test for seroconversion)

335
Q

CI to medical abortion

A
  • Suspected ectopic pregnancy
  • Medical problems
    Severe asthma or COPD; CVD/prosthesis; hepatic failure (risk of bleeding); chronic renal failure (mifepristone can cause electrolyte imbalance); Haemorrhagic disorders and treatment with anticoagulants; Long-term steriod therapy

Known allergy to Mifepristone

336
Q

Complications of abortion

A
Haemorrhage
Uterine perforation 
Cervical Tears: RF -  Young < 18 yr/higher gestation (multipara)
Failure (continuing pregnancy) 
Post abortion Sepsis
Psychological Trauma
337
Q

Reasons for abortion >24 weeks

A
  • women’s life in danger
  • prevent mental/physical injury
  • risk child born serious physical/ mental disability
338
Q

Aftercare after abortion

A
  • Anti-D
  • Metronidazole + doxycycline
  • advise about complications
  • no sex/tampons for 2 weeks
  • future contraceptive plans
  • where to go for counselling
  • VTE risk +/- STI screen